Slot System
Featured Buckets
Featured Buckets Admin

Are serum troponin levels elevated in conditions other than acute coronary syndrome?

Article Type
Changed
Mon, 04/02/2018 - 07:59
Display Headline
Are serum troponin levels elevated in conditions other than acute coronary syndrome?

Yes. Sepsis, stroke, chronic kidney disease, pulmonary disease, chemotherapy, heart failure, and stress cardiomyopathy can all raise serum troponin concentrations, and in some cases the elevations are prognostically important. Careful clinical assessment, serial monitoring of troponin levels, and other supportive tests are usually necessary to tell whether troponin elevations are due to acute coronary syndrome or to these other causes.

NOT ONLY A MARKER OF MYOCARDIAL DAMAGE

Troponin, an intracellular protein found in skeletal and cardiac muscle cells, is essential for muscle contraction. Troponin T and troponin I are clinically equivalent, and both are biomarkers of myocardial damage.

A troponin assay is ordered when patients present with sudden onset of symptoms of acute coronary syndrome such as chest pain, dyspnea, diaphoresis, and electrocardiographic abnormalities. The assay is positive when the manufacturer-specified threshold corresponding to a concentration above the 99th percentile is detected.

Serial testing of serum biomarkers of acute myocardial damage is essential to confirm the diagnosis of myocardial infarction. Because of their higher sensitivity and specificity compared with creatine kinase-MB and other markers, troponins are the preferred biomarker in diagnosing acute coronary syndrome.

In 1984, Piper et al1 reported that free cytosolic pools of cardiac enzymes could be released after reversible myocardial injury as a result of temporary disruption of the cell membrane. This upended the previous assumption that troponin was released only after irreversible myocardial necrosis, and it provided an explanation for troponin elevations observed in conditions with no evidence of epicardial coronary artery disease or permanent myocardial damage.1

SEPSIS

Studies of patients with sepsis, severe sepsis, and septic shock have shown troponin elevations with no evidence of acute coronary syndrome.2 In sepsis, troponin elevations are presumed to be caused by a combination of events. Renal dysfunction leads to decreased clearance of troponin fragments by the kidneys. The massive inflammatory response in septic shock results in cytokine-induced cardiac damage, and increased levels of endogen­ous and exogenous catecholamines damage cardiac myocytes.3

Studies of the prognostic value of these elevations have produced mixed and contradictory results. But a 2013 meta-analysis4 showed that patients with a troponin elevation at the time of diagnosis of sepsis had a risk of death almost twice that of patients without a troponin elevation (relative risk 1.91, 95% confidence interval [CI] 1.63–2.24).

STROKE

Acute ischemic stroke can trigger troponin elevations in several ways. Since the risk factors for acute ischemic stroke and coronary stenosis are similar, patients who have an ischemic stroke have a higher risk of coronary atherosclerosis and coronary stenosis than the general population.5

Stroke can cause a variety of cardiovascular and respiratory responses (eg, tachyarrhythmia, hypertensive crisis, respiratory failure) that increase the stress on the myocardium. In patients with stroke and concurrent coronary artery stenosis, the increased metabolic demand can exceed the oxygen supply capacity, resulting in myocardial ischemia, which can manifest as increased levels of serum troponin.5

Stroke can also cause troponin elevation through neurogenic myocardial damage. Ischemic stroke and intracranial hemorrhage can trigger alterations in autonomic control. Sometimes this results in increased sympathetic activity with concomitant catecholamine surge, leading to contraction band necrosis and other forms of myocardial damage and, as a result, troponin elevation.5,6 This may explain troponin elevation in patients with acute ischemic stroke in the absence of concomitant coronary artery disease. Recent evidence suggests that patients with acute ischemic stroke and elevated troponin had significantly less angiographic evidence of coronary artery disease than matched patients with non-ST-elevation myocardial infarction.7

 

 

CHRONIC KIDNEY DISEASE

Cardiac troponins may be elevated in chronic kidney disease. Explanations for this include the theory that troponin is broken down into fragments that are cleared by the kidney.8 Therefore, decreased renal function leads to an increase in troponin fragments measured with troponin assays. Other explanations are chronic volume overload, chronic elevation of proinflammatory cytokines, and associated comorbidities such as hypertension.

Troponin elevations can have prognostic significance in chronic kidney disease. In a meta-analysis of 98 studies of patients with chronic kidney disease and no symptoms of acute coronary syndrome, troponin elevation was associated with 2- to 4-fold higher rates of all-cause mortality, cardiovascular mortality, and major acute coronary events in both dialysis-dependent and nondialysis patients.8 Thus, troponin is a unique factor in risk-stratification in patients with chronic kidney disease and could affect how it is managed in the future.

To determine if an acute coronary syndrome is taking place when evaluating patients with chronic kidney disease and elevated troponins, physicians must use other evidence—for example, serial measurements of troponin levels showing continued troponin elevation, elevations in troponin from the patient’s baseline, elevated creatine kinase-MB levels, electrocardiographic changes, and clinical symptoms.

PULMONARY DISEASE

Troponin elevation can signify right heart strain in a variety of pulmonary diseases.

Pulmonary embolism. Troponin elevation is a marker of right ventricular dysfunction in patients with moderate to large pulmonary embolism.

In a study of normotensive patients with acute pulmonary embolism, 52% had elevated serum troponin, and they had a higher risk of an adverse outcome (death, recurrent pulmonary embolism, or major bleeding) within 30 days (odds ratio 4.97, 95% CI 1.71–14.43) and a lower probability of 6-month survival.9 Troponin elevation in pulmonary embolism is not helpful in confirming the diagnosis but is primarily useful in prognosis.

Pulmonary arterial hypertension. Cardiac troponin elevations can also indicate severe disease and poor outcomes in patients with pulmonary arterial hypertension. A study by Heresi et al10 confirmed this, even in patients with only slight elevations in troponin levels. Troponin was detected in 17 (25%) of 68 patients with pulmonary arterial hypertension diagnostic category 1. Further, patients with detectable troponin had more advanced functional class symptoms, a shorter 6-minute walk distance, more pericardial effusions, larger right atrial area, and higher B-type natriuretic peptide and C-reactive protein levels.10

Measuring troponins in the setting of pulmonary hypertension allows clinicians to identify high-risk patients and may help guide the management of these patients.

Chronic obstructive pulmonary disease. Elevation of serum troponins is also reported in patients with acute exacerbation of chronic obstructive pulmonary disease and has been correlated with increased all-cause mortality rates in these patients.11

CHEMOTHERAPY

Chemotherapy-induced cardiotoxicity may result in troponin elevations. Chemotherapy causes cardiac toxicity by several mechanisms, including production of oxygen free radicals, disturbance of mitochondrial energy metabolism, intracellular calcium overload, and increased lipid peroxidation. Chemotherapeutic agents associated with cardiotoxicity include anthracyclines, trastuzumab, chlormethine, and mitomycin.

Chemotherapy-induced left ventricular deterioration is often irreversible. Monitoring troponin levels can help identify problems before cardiac dysfunction becomes clinically evident during the weeks and months after the start of high-dose chemotherapy.

Cardinale et al12 found marked myocardial depression 7 months after the start of high-dose chemotherapy. They reported a close relationship between short-term troponin elevation and the greatest reduction in left-ventricular ejection fraction (r = −0.87; P < .0001). Normal troponin values after high-dose chemotherapy also seemed to identify patients at lower risk, with either no cardiac damage or only transient subclinical left-ventricular dysfunction.12

HEART FAILURE

Heart failure leads to release of cardiac troponins through myocardial strain and myocardial death. Volume and pressure overload of the ventricles causes excessive wall tension, resulting in myofibrillar damage. Measuring troponins is an effective way to detect cardiac myolysis in heart failure, independent of the presence of coronary artery disease.

In heart failure, elevated troponins correlate with adverse outcome in both hospitalized and stable patients. In addition, elevation of both troponins and B-type natriuretic peptide is associated with worse heart failure outcomes than elevation of either marker alone.

A prospective study13 of patients with New York Heart Association class III or IV heart failure showed that an increase in troponin concentration from normal baseline was associated with a risk of death, cardiac transplant, or hospitalization that was 3.4 to 5.09 times higher. Further elevations in B-type natriuretic peptide during the study period were associated with a poor outcome (hazard ratio 5.09; P < .001). Combined elevations of troponin and B-type natriuretic peptide defined the group at highest risk (hazard ratio 8.58; P < .001).

Increased myocardial wall stress may lead to decreased subendocardial perfusion, with resulting troponin elevation and decline in left ventricular systolic function. Further, in vitro experiments with myocytes established a link between myocardial wall stretch and programmed cell death, which may contribute to troponin elevations.14

 

 

STRESS CARDIOMYOPATHY

Profound reversible myocardial depression and troponin elevation are seen after sudden emotional stress, a condition called stress-induced or takotsubo cardiomyopathy. While the exact mechanism of stress-induced cardiomyopathy remains unclear, it is thought to be due to sudden supraphysiologic elevation of catecholamines and related neuropeptides. Although vasospasm in the epicardial and microvascular circulation has been suggested as the possible mechanism of left ventricular systolic dysfunction and troponin elevation, cardiac myocyte injury from catecholamine- induced cyclic AMP-mediated calcium overload and oxygen-derived free radicals appears to be a more likely mechanism.15

PSEUDOELEVATIONS OF TROPONIN

In rare cases, endogenous antibodies (eg, heterophilic antibodies) in the blood specimen can interfere with the processing of the troponin immunoassay in the laboratory, causing a false-positive assay. This can occur with samples from patients with a viral infection or autoimmune condition as well as with samples from patients treated with intravenous immunoglobulin (Ig). Heterophilic antibodies can bind to the Fc region of the test antibodies in certain troponin assays, leading to false-positive elevations.16 Macrotroponin, a molecule found in patients with autoantibodies against troponin I, is composed of troponin I fragments and IgG antibodies and can also cause a false-positive troponin immunoassay.16

In patients with seropositive rheumatoid arthritis, a false-positive troponin I assay was associated with a high concentration of IgM rheumatoid factor with the use of certain immunoassay techniques.17 In patients with acute skeletal muscle injury, the first-generation troponin T assay was found to be falsely positive due to the nonspecific binding of skeletal muscle troponin T to the walls of the test tube used for the assay. When the second-generation troponin T assay was used, troponin T levels were found to be slightly more positive than troponin I levels (1.7 vs 1.5 times the upper limit of normal), especially in patients with renal failure.18

Troponin may also be falsely elevated in hemolyzed blood samples. This has to be taken into consideration in interpreting the results of troponin testing in severely hemolyzed blood samples. However, Puelacher et al19 suggested that the presence of hemolysis did not appear to interfere with clinical value of the test.

References
  1. Piper HM, Schwartz P, Spahr R, Hütter J, Spieckermann P. Early enzyme release from myocardial cells is not due to irreversible cell damage. J Mol Cell Cardiol 1984; 16(4):385–388. doi:10.1016/S0022-2828(84)80609-4
  2. Ammann P, Fehr T, Minder EI, Günter C, Bertel O. Elevation of troponin I in sepsis and septic shock. Intensive Care Med 2001; 27(6):965–969.
  3. Landesberg G, Jaffe AS, Gilon D, et al. Troponin elevation in severe sepsis and septic shock. Crit Care Med 2014; 42(4):790–800. doi:10.1097/CCM.0000000000000107
  4. Bessière F, Khenifer S, Dubourg J, Durieu I, Lega JC. Prognostic value of troponins in sepsis: a meta-analysis. Intensive Care Med 2013; 39(7):1181–1189. doi:10.1007/s00134-013-2902-3
  5. Scheitz JF, Nolte CH, Laufs U, Endres M. Application and interpretation of high-sensitivity cardiac troponin assays in patients with acute ischemic stroke. Stroke 2015; 46(4):1132–1140. doi:10.1161/STROKEAHA.114.007858
  6. Naidech AM, Kreiter KT, Janjua N, et al. Cardiac troponin elevation, cardiovascular morbidity, and outcome after subarachnoid hemorrhage. Circulation 2005; 112(18):2851–2656. doi:10.1161/CIRCULATIONAHA.105.533620
  7. Mochmann HC, Scheitz JF, Petzold GC, et al; TRELAS Study Group. Coronary angiographic findings in acute ischemic stroke patients with elevated cardiac troponin: the Troponin Elevation in Acute Ischemic Stroke (TRELAS) Study. Circulation 2016; 133(13):1264–1271. doi:10.1161/CIRCULATIONAHA.115.018547
  8. Michos ED, Wilson LM, Yeh HC, et al. Prognostic value of cardiac troponin in patients with chronic kidney disease without suspected acute coronary syndrome. Ann Intern Med 2014; 161(7):491–501. doi:10.7326/M14-0743
  9. Lankeit M, Jiménez D, Kostrubiec M, et al. Predictive value of the high-sensitivity troponin T assay and the simplified pulmonary embolism severity index in hemodynamically stable patients with acute pulmonary embolism: a prospective validation study. Circulation 2011; 124(24):2716–2724. doi:10.1161/CIRCULATIONAHA.111.051177
  10. Heresi GA, Tang WH, Aytekin M, Hammel J, Hazen SL, Dweik RA. Sensitive cardiac troponin I predicts poor outcomes in pulmonary arterial hypertension. Eur Respir J 2012; 39(4)939–944. doi:10.1183/09031936.00067011
  11. Pavasini R, d’Ascenzo F, Campo G, et al. Cardiac troponin elevation predicts all-cause mortality in patients with acute exacerbation of chronic obstructive pulmonary disease: systematic review and meta-analysis. Int J Cardiol 2015; 191:187–193. doi:10.1016/j.ijcard.2015.05.006
  12. Cardinale D, Sandri MT, Martinoni A, et al. Left ventricular dysfunction predicted by early troponin I release after high-dose chemotherapy. J Am Coll Cardiol 2000; 36(2):517–522.
  13. Miller WL, Hartman KA, Burritt MF, et al. Serial biomarker measurements in ambulatory patients with chronic heart failure: the importance of change over time. Circulation 2007; 116(3):249–257. doi:10.1161/CIRCULATIONAHA.107.694562
  14. Logeart D, Beyne P, Cusson C, et al. Evidence of cardiac myolysis in severe nonischemic heart failure and the potential role of increased wall strain. Am Heart J 2001; 141(2):247–253. doi:10.1067/mhj.2001.111767
  15. Whittstein IS, Thiemann DR, Lima JA, et al. Neurohumoral features of myocardial stunning due to sudden emotional stress. N Engl J Med 2005; 352(6):539–548. doi:10.1056/NEJMoa043046
  16. McClennen S, Halamka JD, Horowitz GL, Kannam JP, Ho KK. Clinical prevalence and ramifications of false-positive cardiac troponin I elevations from the Abbott AxSYM Analyzer. Am J Cardiol 2003; 91(9):1125–1127.
  17. Bradham WS, Bian A, Oeser A, et al. High-sensitivity cardiac troponin-I is elevated in patients with rheumatoid arthritis, independent of cardiovascular risk factors and inflammation. PLoS One 2012; 7(6):e38930. doi:10.1371/journal.pone.0038930
  18. Li SF, Zapata J, Tillem E. The prevalence of false-positive cardiac troponin I in ED patients with rhabdomyolysis. Am J Emerg Med 2005; 23(7):860–863. doi:10.1016/j.ajem.2005.05.008
  19. Puelacher C, Twerenbold R, Mosimann T, et al. Effects of hemolysis on the diagnostic accuracy of cardiac troponin I for the diagnosis of myocardial infarction. Int J Cardiol 2015; 187:313–315. doi:10.1016/j.ijcard.2015.03.378
Article PDF
Author and Disclosure Information

Kiran Sebastian, MD
Division of Pulmonary, Critical Care, and Sleep Medicine, Wayne State University School of Medicine, Detroit, MI

Alexander Wester, BS
Division of Pulmonary, Critical Care, and Sleep Medicine, Wayne State University School of Medicine, Detroit, MI

Anupama Kottam, MD
Division of Cardiology, Wayne State University School of Medicine, Detroit, MI

Ayman O. Soubani, MD
Division of Pulmonary, Critical Care, and Sleep Medicine, Wayne State University School of Medicine; Professor of Medicine, Wayne State University School of Medicine; Medical Director, Medical ICU, Harper University Hospital; Service Chief, Pulmonary and Critical Care, and Medical Director, Critical Care Service, Karmanos Cancer Center, Detroit, MI

Address: Ayman O. Soubani, MD, Professor of Medicine, Wayne State University School of Medicine, 3990 John R-3 Hudson, Detroit, MI 48201; asoubani@med.wayne.edu

Issue
Cleveland Clinic Journal of Medicine - 85(4)
Publications
Topics
Page Number
274-277
Legacy Keywords
troponin, laboratory testing, acute coronary syndrome, myocardial infarction, MI, sepsis, stroke, chronic kidney disease, CKD, heart failure, stress cardiomyopathy, Kiran Sebastian, Alexander Wester, Anapama Kottam, Ayman Soubani
Sections
Author and Disclosure Information

Kiran Sebastian, MD
Division of Pulmonary, Critical Care, and Sleep Medicine, Wayne State University School of Medicine, Detroit, MI

Alexander Wester, BS
Division of Pulmonary, Critical Care, and Sleep Medicine, Wayne State University School of Medicine, Detroit, MI

Anupama Kottam, MD
Division of Cardiology, Wayne State University School of Medicine, Detroit, MI

Ayman O. Soubani, MD
Division of Pulmonary, Critical Care, and Sleep Medicine, Wayne State University School of Medicine; Professor of Medicine, Wayne State University School of Medicine; Medical Director, Medical ICU, Harper University Hospital; Service Chief, Pulmonary and Critical Care, and Medical Director, Critical Care Service, Karmanos Cancer Center, Detroit, MI

Address: Ayman O. Soubani, MD, Professor of Medicine, Wayne State University School of Medicine, 3990 John R-3 Hudson, Detroit, MI 48201; asoubani@med.wayne.edu

Author and Disclosure Information

Kiran Sebastian, MD
Division of Pulmonary, Critical Care, and Sleep Medicine, Wayne State University School of Medicine, Detroit, MI

Alexander Wester, BS
Division of Pulmonary, Critical Care, and Sleep Medicine, Wayne State University School of Medicine, Detroit, MI

Anupama Kottam, MD
Division of Cardiology, Wayne State University School of Medicine, Detroit, MI

Ayman O. Soubani, MD
Division of Pulmonary, Critical Care, and Sleep Medicine, Wayne State University School of Medicine; Professor of Medicine, Wayne State University School of Medicine; Medical Director, Medical ICU, Harper University Hospital; Service Chief, Pulmonary and Critical Care, and Medical Director, Critical Care Service, Karmanos Cancer Center, Detroit, MI

Address: Ayman O. Soubani, MD, Professor of Medicine, Wayne State University School of Medicine, 3990 John R-3 Hudson, Detroit, MI 48201; asoubani@med.wayne.edu

Article PDF
Article PDF
Related Articles

Yes. Sepsis, stroke, chronic kidney disease, pulmonary disease, chemotherapy, heart failure, and stress cardiomyopathy can all raise serum troponin concentrations, and in some cases the elevations are prognostically important. Careful clinical assessment, serial monitoring of troponin levels, and other supportive tests are usually necessary to tell whether troponin elevations are due to acute coronary syndrome or to these other causes.

NOT ONLY A MARKER OF MYOCARDIAL DAMAGE

Troponin, an intracellular protein found in skeletal and cardiac muscle cells, is essential for muscle contraction. Troponin T and troponin I are clinically equivalent, and both are biomarkers of myocardial damage.

A troponin assay is ordered when patients present with sudden onset of symptoms of acute coronary syndrome such as chest pain, dyspnea, diaphoresis, and electrocardiographic abnormalities. The assay is positive when the manufacturer-specified threshold corresponding to a concentration above the 99th percentile is detected.

Serial testing of serum biomarkers of acute myocardial damage is essential to confirm the diagnosis of myocardial infarction. Because of their higher sensitivity and specificity compared with creatine kinase-MB and other markers, troponins are the preferred biomarker in diagnosing acute coronary syndrome.

In 1984, Piper et al1 reported that free cytosolic pools of cardiac enzymes could be released after reversible myocardial injury as a result of temporary disruption of the cell membrane. This upended the previous assumption that troponin was released only after irreversible myocardial necrosis, and it provided an explanation for troponin elevations observed in conditions with no evidence of epicardial coronary artery disease or permanent myocardial damage.1

SEPSIS

Studies of patients with sepsis, severe sepsis, and septic shock have shown troponin elevations with no evidence of acute coronary syndrome.2 In sepsis, troponin elevations are presumed to be caused by a combination of events. Renal dysfunction leads to decreased clearance of troponin fragments by the kidneys. The massive inflammatory response in septic shock results in cytokine-induced cardiac damage, and increased levels of endogen­ous and exogenous catecholamines damage cardiac myocytes.3

Studies of the prognostic value of these elevations have produced mixed and contradictory results. But a 2013 meta-analysis4 showed that patients with a troponin elevation at the time of diagnosis of sepsis had a risk of death almost twice that of patients without a troponin elevation (relative risk 1.91, 95% confidence interval [CI] 1.63–2.24).

STROKE

Acute ischemic stroke can trigger troponin elevations in several ways. Since the risk factors for acute ischemic stroke and coronary stenosis are similar, patients who have an ischemic stroke have a higher risk of coronary atherosclerosis and coronary stenosis than the general population.5

Stroke can cause a variety of cardiovascular and respiratory responses (eg, tachyarrhythmia, hypertensive crisis, respiratory failure) that increase the stress on the myocardium. In patients with stroke and concurrent coronary artery stenosis, the increased metabolic demand can exceed the oxygen supply capacity, resulting in myocardial ischemia, which can manifest as increased levels of serum troponin.5

Stroke can also cause troponin elevation through neurogenic myocardial damage. Ischemic stroke and intracranial hemorrhage can trigger alterations in autonomic control. Sometimes this results in increased sympathetic activity with concomitant catecholamine surge, leading to contraction band necrosis and other forms of myocardial damage and, as a result, troponin elevation.5,6 This may explain troponin elevation in patients with acute ischemic stroke in the absence of concomitant coronary artery disease. Recent evidence suggests that patients with acute ischemic stroke and elevated troponin had significantly less angiographic evidence of coronary artery disease than matched patients with non-ST-elevation myocardial infarction.7

 

 

CHRONIC KIDNEY DISEASE

Cardiac troponins may be elevated in chronic kidney disease. Explanations for this include the theory that troponin is broken down into fragments that are cleared by the kidney.8 Therefore, decreased renal function leads to an increase in troponin fragments measured with troponin assays. Other explanations are chronic volume overload, chronic elevation of proinflammatory cytokines, and associated comorbidities such as hypertension.

Troponin elevations can have prognostic significance in chronic kidney disease. In a meta-analysis of 98 studies of patients with chronic kidney disease and no symptoms of acute coronary syndrome, troponin elevation was associated with 2- to 4-fold higher rates of all-cause mortality, cardiovascular mortality, and major acute coronary events in both dialysis-dependent and nondialysis patients.8 Thus, troponin is a unique factor in risk-stratification in patients with chronic kidney disease and could affect how it is managed in the future.

To determine if an acute coronary syndrome is taking place when evaluating patients with chronic kidney disease and elevated troponins, physicians must use other evidence—for example, serial measurements of troponin levels showing continued troponin elevation, elevations in troponin from the patient’s baseline, elevated creatine kinase-MB levels, electrocardiographic changes, and clinical symptoms.

PULMONARY DISEASE

Troponin elevation can signify right heart strain in a variety of pulmonary diseases.

Pulmonary embolism. Troponin elevation is a marker of right ventricular dysfunction in patients with moderate to large pulmonary embolism.

In a study of normotensive patients with acute pulmonary embolism, 52% had elevated serum troponin, and they had a higher risk of an adverse outcome (death, recurrent pulmonary embolism, or major bleeding) within 30 days (odds ratio 4.97, 95% CI 1.71–14.43) and a lower probability of 6-month survival.9 Troponin elevation in pulmonary embolism is not helpful in confirming the diagnosis but is primarily useful in prognosis.

Pulmonary arterial hypertension. Cardiac troponin elevations can also indicate severe disease and poor outcomes in patients with pulmonary arterial hypertension. A study by Heresi et al10 confirmed this, even in patients with only slight elevations in troponin levels. Troponin was detected in 17 (25%) of 68 patients with pulmonary arterial hypertension diagnostic category 1. Further, patients with detectable troponin had more advanced functional class symptoms, a shorter 6-minute walk distance, more pericardial effusions, larger right atrial area, and higher B-type natriuretic peptide and C-reactive protein levels.10

Measuring troponins in the setting of pulmonary hypertension allows clinicians to identify high-risk patients and may help guide the management of these patients.

Chronic obstructive pulmonary disease. Elevation of serum troponins is also reported in patients with acute exacerbation of chronic obstructive pulmonary disease and has been correlated with increased all-cause mortality rates in these patients.11

CHEMOTHERAPY

Chemotherapy-induced cardiotoxicity may result in troponin elevations. Chemotherapy causes cardiac toxicity by several mechanisms, including production of oxygen free radicals, disturbance of mitochondrial energy metabolism, intracellular calcium overload, and increased lipid peroxidation. Chemotherapeutic agents associated with cardiotoxicity include anthracyclines, trastuzumab, chlormethine, and mitomycin.

Chemotherapy-induced left ventricular deterioration is often irreversible. Monitoring troponin levels can help identify problems before cardiac dysfunction becomes clinically evident during the weeks and months after the start of high-dose chemotherapy.

Cardinale et al12 found marked myocardial depression 7 months after the start of high-dose chemotherapy. They reported a close relationship between short-term troponin elevation and the greatest reduction in left-ventricular ejection fraction (r = −0.87; P < .0001). Normal troponin values after high-dose chemotherapy also seemed to identify patients at lower risk, with either no cardiac damage or only transient subclinical left-ventricular dysfunction.12

HEART FAILURE

Heart failure leads to release of cardiac troponins through myocardial strain and myocardial death. Volume and pressure overload of the ventricles causes excessive wall tension, resulting in myofibrillar damage. Measuring troponins is an effective way to detect cardiac myolysis in heart failure, independent of the presence of coronary artery disease.

In heart failure, elevated troponins correlate with adverse outcome in both hospitalized and stable patients. In addition, elevation of both troponins and B-type natriuretic peptide is associated with worse heart failure outcomes than elevation of either marker alone.

A prospective study13 of patients with New York Heart Association class III or IV heart failure showed that an increase in troponin concentration from normal baseline was associated with a risk of death, cardiac transplant, or hospitalization that was 3.4 to 5.09 times higher. Further elevations in B-type natriuretic peptide during the study period were associated with a poor outcome (hazard ratio 5.09; P < .001). Combined elevations of troponin and B-type natriuretic peptide defined the group at highest risk (hazard ratio 8.58; P < .001).

Increased myocardial wall stress may lead to decreased subendocardial perfusion, with resulting troponin elevation and decline in left ventricular systolic function. Further, in vitro experiments with myocytes established a link between myocardial wall stretch and programmed cell death, which may contribute to troponin elevations.14

 

 

STRESS CARDIOMYOPATHY

Profound reversible myocardial depression and troponin elevation are seen after sudden emotional stress, a condition called stress-induced or takotsubo cardiomyopathy. While the exact mechanism of stress-induced cardiomyopathy remains unclear, it is thought to be due to sudden supraphysiologic elevation of catecholamines and related neuropeptides. Although vasospasm in the epicardial and microvascular circulation has been suggested as the possible mechanism of left ventricular systolic dysfunction and troponin elevation, cardiac myocyte injury from catecholamine- induced cyclic AMP-mediated calcium overload and oxygen-derived free radicals appears to be a more likely mechanism.15

PSEUDOELEVATIONS OF TROPONIN

In rare cases, endogenous antibodies (eg, heterophilic antibodies) in the blood specimen can interfere with the processing of the troponin immunoassay in the laboratory, causing a false-positive assay. This can occur with samples from patients with a viral infection or autoimmune condition as well as with samples from patients treated with intravenous immunoglobulin (Ig). Heterophilic antibodies can bind to the Fc region of the test antibodies in certain troponin assays, leading to false-positive elevations.16 Macrotroponin, a molecule found in patients with autoantibodies against troponin I, is composed of troponin I fragments and IgG antibodies and can also cause a false-positive troponin immunoassay.16

In patients with seropositive rheumatoid arthritis, a false-positive troponin I assay was associated with a high concentration of IgM rheumatoid factor with the use of certain immunoassay techniques.17 In patients with acute skeletal muscle injury, the first-generation troponin T assay was found to be falsely positive due to the nonspecific binding of skeletal muscle troponin T to the walls of the test tube used for the assay. When the second-generation troponin T assay was used, troponin T levels were found to be slightly more positive than troponin I levels (1.7 vs 1.5 times the upper limit of normal), especially in patients with renal failure.18

Troponin may also be falsely elevated in hemolyzed blood samples. This has to be taken into consideration in interpreting the results of troponin testing in severely hemolyzed blood samples. However, Puelacher et al19 suggested that the presence of hemolysis did not appear to interfere with clinical value of the test.

Yes. Sepsis, stroke, chronic kidney disease, pulmonary disease, chemotherapy, heart failure, and stress cardiomyopathy can all raise serum troponin concentrations, and in some cases the elevations are prognostically important. Careful clinical assessment, serial monitoring of troponin levels, and other supportive tests are usually necessary to tell whether troponin elevations are due to acute coronary syndrome or to these other causes.

NOT ONLY A MARKER OF MYOCARDIAL DAMAGE

Troponin, an intracellular protein found in skeletal and cardiac muscle cells, is essential for muscle contraction. Troponin T and troponin I are clinically equivalent, and both are biomarkers of myocardial damage.

A troponin assay is ordered when patients present with sudden onset of symptoms of acute coronary syndrome such as chest pain, dyspnea, diaphoresis, and electrocardiographic abnormalities. The assay is positive when the manufacturer-specified threshold corresponding to a concentration above the 99th percentile is detected.

Serial testing of serum biomarkers of acute myocardial damage is essential to confirm the diagnosis of myocardial infarction. Because of their higher sensitivity and specificity compared with creatine kinase-MB and other markers, troponins are the preferred biomarker in diagnosing acute coronary syndrome.

In 1984, Piper et al1 reported that free cytosolic pools of cardiac enzymes could be released after reversible myocardial injury as a result of temporary disruption of the cell membrane. This upended the previous assumption that troponin was released only after irreversible myocardial necrosis, and it provided an explanation for troponin elevations observed in conditions with no evidence of epicardial coronary artery disease or permanent myocardial damage.1

SEPSIS

Studies of patients with sepsis, severe sepsis, and septic shock have shown troponin elevations with no evidence of acute coronary syndrome.2 In sepsis, troponin elevations are presumed to be caused by a combination of events. Renal dysfunction leads to decreased clearance of troponin fragments by the kidneys. The massive inflammatory response in septic shock results in cytokine-induced cardiac damage, and increased levels of endogen­ous and exogenous catecholamines damage cardiac myocytes.3

Studies of the prognostic value of these elevations have produced mixed and contradictory results. But a 2013 meta-analysis4 showed that patients with a troponin elevation at the time of diagnosis of sepsis had a risk of death almost twice that of patients without a troponin elevation (relative risk 1.91, 95% confidence interval [CI] 1.63–2.24).

STROKE

Acute ischemic stroke can trigger troponin elevations in several ways. Since the risk factors for acute ischemic stroke and coronary stenosis are similar, patients who have an ischemic stroke have a higher risk of coronary atherosclerosis and coronary stenosis than the general population.5

Stroke can cause a variety of cardiovascular and respiratory responses (eg, tachyarrhythmia, hypertensive crisis, respiratory failure) that increase the stress on the myocardium. In patients with stroke and concurrent coronary artery stenosis, the increased metabolic demand can exceed the oxygen supply capacity, resulting in myocardial ischemia, which can manifest as increased levels of serum troponin.5

Stroke can also cause troponin elevation through neurogenic myocardial damage. Ischemic stroke and intracranial hemorrhage can trigger alterations in autonomic control. Sometimes this results in increased sympathetic activity with concomitant catecholamine surge, leading to contraction band necrosis and other forms of myocardial damage and, as a result, troponin elevation.5,6 This may explain troponin elevation in patients with acute ischemic stroke in the absence of concomitant coronary artery disease. Recent evidence suggests that patients with acute ischemic stroke and elevated troponin had significantly less angiographic evidence of coronary artery disease than matched patients with non-ST-elevation myocardial infarction.7

 

 

CHRONIC KIDNEY DISEASE

Cardiac troponins may be elevated in chronic kidney disease. Explanations for this include the theory that troponin is broken down into fragments that are cleared by the kidney.8 Therefore, decreased renal function leads to an increase in troponin fragments measured with troponin assays. Other explanations are chronic volume overload, chronic elevation of proinflammatory cytokines, and associated comorbidities such as hypertension.

Troponin elevations can have prognostic significance in chronic kidney disease. In a meta-analysis of 98 studies of patients with chronic kidney disease and no symptoms of acute coronary syndrome, troponin elevation was associated with 2- to 4-fold higher rates of all-cause mortality, cardiovascular mortality, and major acute coronary events in both dialysis-dependent and nondialysis patients.8 Thus, troponin is a unique factor in risk-stratification in patients with chronic kidney disease and could affect how it is managed in the future.

To determine if an acute coronary syndrome is taking place when evaluating patients with chronic kidney disease and elevated troponins, physicians must use other evidence—for example, serial measurements of troponin levels showing continued troponin elevation, elevations in troponin from the patient’s baseline, elevated creatine kinase-MB levels, electrocardiographic changes, and clinical symptoms.

PULMONARY DISEASE

Troponin elevation can signify right heart strain in a variety of pulmonary diseases.

Pulmonary embolism. Troponin elevation is a marker of right ventricular dysfunction in patients with moderate to large pulmonary embolism.

In a study of normotensive patients with acute pulmonary embolism, 52% had elevated serum troponin, and they had a higher risk of an adverse outcome (death, recurrent pulmonary embolism, or major bleeding) within 30 days (odds ratio 4.97, 95% CI 1.71–14.43) and a lower probability of 6-month survival.9 Troponin elevation in pulmonary embolism is not helpful in confirming the diagnosis but is primarily useful in prognosis.

Pulmonary arterial hypertension. Cardiac troponin elevations can also indicate severe disease and poor outcomes in patients with pulmonary arterial hypertension. A study by Heresi et al10 confirmed this, even in patients with only slight elevations in troponin levels. Troponin was detected in 17 (25%) of 68 patients with pulmonary arterial hypertension diagnostic category 1. Further, patients with detectable troponin had more advanced functional class symptoms, a shorter 6-minute walk distance, more pericardial effusions, larger right atrial area, and higher B-type natriuretic peptide and C-reactive protein levels.10

Measuring troponins in the setting of pulmonary hypertension allows clinicians to identify high-risk patients and may help guide the management of these patients.

Chronic obstructive pulmonary disease. Elevation of serum troponins is also reported in patients with acute exacerbation of chronic obstructive pulmonary disease and has been correlated with increased all-cause mortality rates in these patients.11

CHEMOTHERAPY

Chemotherapy-induced cardiotoxicity may result in troponin elevations. Chemotherapy causes cardiac toxicity by several mechanisms, including production of oxygen free radicals, disturbance of mitochondrial energy metabolism, intracellular calcium overload, and increased lipid peroxidation. Chemotherapeutic agents associated with cardiotoxicity include anthracyclines, trastuzumab, chlormethine, and mitomycin.

Chemotherapy-induced left ventricular deterioration is often irreversible. Monitoring troponin levels can help identify problems before cardiac dysfunction becomes clinically evident during the weeks and months after the start of high-dose chemotherapy.

Cardinale et al12 found marked myocardial depression 7 months after the start of high-dose chemotherapy. They reported a close relationship between short-term troponin elevation and the greatest reduction in left-ventricular ejection fraction (r = −0.87; P < .0001). Normal troponin values after high-dose chemotherapy also seemed to identify patients at lower risk, with either no cardiac damage or only transient subclinical left-ventricular dysfunction.12

HEART FAILURE

Heart failure leads to release of cardiac troponins through myocardial strain and myocardial death. Volume and pressure overload of the ventricles causes excessive wall tension, resulting in myofibrillar damage. Measuring troponins is an effective way to detect cardiac myolysis in heart failure, independent of the presence of coronary artery disease.

In heart failure, elevated troponins correlate with adverse outcome in both hospitalized and stable patients. In addition, elevation of both troponins and B-type natriuretic peptide is associated with worse heart failure outcomes than elevation of either marker alone.

A prospective study13 of patients with New York Heart Association class III or IV heart failure showed that an increase in troponin concentration from normal baseline was associated with a risk of death, cardiac transplant, or hospitalization that was 3.4 to 5.09 times higher. Further elevations in B-type natriuretic peptide during the study period were associated with a poor outcome (hazard ratio 5.09; P < .001). Combined elevations of troponin and B-type natriuretic peptide defined the group at highest risk (hazard ratio 8.58; P < .001).

Increased myocardial wall stress may lead to decreased subendocardial perfusion, with resulting troponin elevation and decline in left ventricular systolic function. Further, in vitro experiments with myocytes established a link between myocardial wall stretch and programmed cell death, which may contribute to troponin elevations.14

 

 

STRESS CARDIOMYOPATHY

Profound reversible myocardial depression and troponin elevation are seen after sudden emotional stress, a condition called stress-induced or takotsubo cardiomyopathy. While the exact mechanism of stress-induced cardiomyopathy remains unclear, it is thought to be due to sudden supraphysiologic elevation of catecholamines and related neuropeptides. Although vasospasm in the epicardial and microvascular circulation has been suggested as the possible mechanism of left ventricular systolic dysfunction and troponin elevation, cardiac myocyte injury from catecholamine- induced cyclic AMP-mediated calcium overload and oxygen-derived free radicals appears to be a more likely mechanism.15

PSEUDOELEVATIONS OF TROPONIN

In rare cases, endogenous antibodies (eg, heterophilic antibodies) in the blood specimen can interfere with the processing of the troponin immunoassay in the laboratory, causing a false-positive assay. This can occur with samples from patients with a viral infection or autoimmune condition as well as with samples from patients treated with intravenous immunoglobulin (Ig). Heterophilic antibodies can bind to the Fc region of the test antibodies in certain troponin assays, leading to false-positive elevations.16 Macrotroponin, a molecule found in patients with autoantibodies against troponin I, is composed of troponin I fragments and IgG antibodies and can also cause a false-positive troponin immunoassay.16

In patients with seropositive rheumatoid arthritis, a false-positive troponin I assay was associated with a high concentration of IgM rheumatoid factor with the use of certain immunoassay techniques.17 In patients with acute skeletal muscle injury, the first-generation troponin T assay was found to be falsely positive due to the nonspecific binding of skeletal muscle troponin T to the walls of the test tube used for the assay. When the second-generation troponin T assay was used, troponin T levels were found to be slightly more positive than troponin I levels (1.7 vs 1.5 times the upper limit of normal), especially in patients with renal failure.18

Troponin may also be falsely elevated in hemolyzed blood samples. This has to be taken into consideration in interpreting the results of troponin testing in severely hemolyzed blood samples. However, Puelacher et al19 suggested that the presence of hemolysis did not appear to interfere with clinical value of the test.

References
  1. Piper HM, Schwartz P, Spahr R, Hütter J, Spieckermann P. Early enzyme release from myocardial cells is not due to irreversible cell damage. J Mol Cell Cardiol 1984; 16(4):385–388. doi:10.1016/S0022-2828(84)80609-4
  2. Ammann P, Fehr T, Minder EI, Günter C, Bertel O. Elevation of troponin I in sepsis and septic shock. Intensive Care Med 2001; 27(6):965–969.
  3. Landesberg G, Jaffe AS, Gilon D, et al. Troponin elevation in severe sepsis and septic shock. Crit Care Med 2014; 42(4):790–800. doi:10.1097/CCM.0000000000000107
  4. Bessière F, Khenifer S, Dubourg J, Durieu I, Lega JC. Prognostic value of troponins in sepsis: a meta-analysis. Intensive Care Med 2013; 39(7):1181–1189. doi:10.1007/s00134-013-2902-3
  5. Scheitz JF, Nolte CH, Laufs U, Endres M. Application and interpretation of high-sensitivity cardiac troponin assays in patients with acute ischemic stroke. Stroke 2015; 46(4):1132–1140. doi:10.1161/STROKEAHA.114.007858
  6. Naidech AM, Kreiter KT, Janjua N, et al. Cardiac troponin elevation, cardiovascular morbidity, and outcome after subarachnoid hemorrhage. Circulation 2005; 112(18):2851–2656. doi:10.1161/CIRCULATIONAHA.105.533620
  7. Mochmann HC, Scheitz JF, Petzold GC, et al; TRELAS Study Group. Coronary angiographic findings in acute ischemic stroke patients with elevated cardiac troponin: the Troponin Elevation in Acute Ischemic Stroke (TRELAS) Study. Circulation 2016; 133(13):1264–1271. doi:10.1161/CIRCULATIONAHA.115.018547
  8. Michos ED, Wilson LM, Yeh HC, et al. Prognostic value of cardiac troponin in patients with chronic kidney disease without suspected acute coronary syndrome. Ann Intern Med 2014; 161(7):491–501. doi:10.7326/M14-0743
  9. Lankeit M, Jiménez D, Kostrubiec M, et al. Predictive value of the high-sensitivity troponin T assay and the simplified pulmonary embolism severity index in hemodynamically stable patients with acute pulmonary embolism: a prospective validation study. Circulation 2011; 124(24):2716–2724. doi:10.1161/CIRCULATIONAHA.111.051177
  10. Heresi GA, Tang WH, Aytekin M, Hammel J, Hazen SL, Dweik RA. Sensitive cardiac troponin I predicts poor outcomes in pulmonary arterial hypertension. Eur Respir J 2012; 39(4)939–944. doi:10.1183/09031936.00067011
  11. Pavasini R, d’Ascenzo F, Campo G, et al. Cardiac troponin elevation predicts all-cause mortality in patients with acute exacerbation of chronic obstructive pulmonary disease: systematic review and meta-analysis. Int J Cardiol 2015; 191:187–193. doi:10.1016/j.ijcard.2015.05.006
  12. Cardinale D, Sandri MT, Martinoni A, et al. Left ventricular dysfunction predicted by early troponin I release after high-dose chemotherapy. J Am Coll Cardiol 2000; 36(2):517–522.
  13. Miller WL, Hartman KA, Burritt MF, et al. Serial biomarker measurements in ambulatory patients with chronic heart failure: the importance of change over time. Circulation 2007; 116(3):249–257. doi:10.1161/CIRCULATIONAHA.107.694562
  14. Logeart D, Beyne P, Cusson C, et al. Evidence of cardiac myolysis in severe nonischemic heart failure and the potential role of increased wall strain. Am Heart J 2001; 141(2):247–253. doi:10.1067/mhj.2001.111767
  15. Whittstein IS, Thiemann DR, Lima JA, et al. Neurohumoral features of myocardial stunning due to sudden emotional stress. N Engl J Med 2005; 352(6):539–548. doi:10.1056/NEJMoa043046
  16. McClennen S, Halamka JD, Horowitz GL, Kannam JP, Ho KK. Clinical prevalence and ramifications of false-positive cardiac troponin I elevations from the Abbott AxSYM Analyzer. Am J Cardiol 2003; 91(9):1125–1127.
  17. Bradham WS, Bian A, Oeser A, et al. High-sensitivity cardiac troponin-I is elevated in patients with rheumatoid arthritis, independent of cardiovascular risk factors and inflammation. PLoS One 2012; 7(6):e38930. doi:10.1371/journal.pone.0038930
  18. Li SF, Zapata J, Tillem E. The prevalence of false-positive cardiac troponin I in ED patients with rhabdomyolysis. Am J Emerg Med 2005; 23(7):860–863. doi:10.1016/j.ajem.2005.05.008
  19. Puelacher C, Twerenbold R, Mosimann T, et al. Effects of hemolysis on the diagnostic accuracy of cardiac troponin I for the diagnosis of myocardial infarction. Int J Cardiol 2015; 187:313–315. doi:10.1016/j.ijcard.2015.03.378
References
  1. Piper HM, Schwartz P, Spahr R, Hütter J, Spieckermann P. Early enzyme release from myocardial cells is not due to irreversible cell damage. J Mol Cell Cardiol 1984; 16(4):385–388. doi:10.1016/S0022-2828(84)80609-4
  2. Ammann P, Fehr T, Minder EI, Günter C, Bertel O. Elevation of troponin I in sepsis and septic shock. Intensive Care Med 2001; 27(6):965–969.
  3. Landesberg G, Jaffe AS, Gilon D, et al. Troponin elevation in severe sepsis and septic shock. Crit Care Med 2014; 42(4):790–800. doi:10.1097/CCM.0000000000000107
  4. Bessière F, Khenifer S, Dubourg J, Durieu I, Lega JC. Prognostic value of troponins in sepsis: a meta-analysis. Intensive Care Med 2013; 39(7):1181–1189. doi:10.1007/s00134-013-2902-3
  5. Scheitz JF, Nolte CH, Laufs U, Endres M. Application and interpretation of high-sensitivity cardiac troponin assays in patients with acute ischemic stroke. Stroke 2015; 46(4):1132–1140. doi:10.1161/STROKEAHA.114.007858
  6. Naidech AM, Kreiter KT, Janjua N, et al. Cardiac troponin elevation, cardiovascular morbidity, and outcome after subarachnoid hemorrhage. Circulation 2005; 112(18):2851–2656. doi:10.1161/CIRCULATIONAHA.105.533620
  7. Mochmann HC, Scheitz JF, Petzold GC, et al; TRELAS Study Group. Coronary angiographic findings in acute ischemic stroke patients with elevated cardiac troponin: the Troponin Elevation in Acute Ischemic Stroke (TRELAS) Study. Circulation 2016; 133(13):1264–1271. doi:10.1161/CIRCULATIONAHA.115.018547
  8. Michos ED, Wilson LM, Yeh HC, et al. Prognostic value of cardiac troponin in patients with chronic kidney disease without suspected acute coronary syndrome. Ann Intern Med 2014; 161(7):491–501. doi:10.7326/M14-0743
  9. Lankeit M, Jiménez D, Kostrubiec M, et al. Predictive value of the high-sensitivity troponin T assay and the simplified pulmonary embolism severity index in hemodynamically stable patients with acute pulmonary embolism: a prospective validation study. Circulation 2011; 124(24):2716–2724. doi:10.1161/CIRCULATIONAHA.111.051177
  10. Heresi GA, Tang WH, Aytekin M, Hammel J, Hazen SL, Dweik RA. Sensitive cardiac troponin I predicts poor outcomes in pulmonary arterial hypertension. Eur Respir J 2012; 39(4)939–944. doi:10.1183/09031936.00067011
  11. Pavasini R, d’Ascenzo F, Campo G, et al. Cardiac troponin elevation predicts all-cause mortality in patients with acute exacerbation of chronic obstructive pulmonary disease: systematic review and meta-analysis. Int J Cardiol 2015; 191:187–193. doi:10.1016/j.ijcard.2015.05.006
  12. Cardinale D, Sandri MT, Martinoni A, et al. Left ventricular dysfunction predicted by early troponin I release after high-dose chemotherapy. J Am Coll Cardiol 2000; 36(2):517–522.
  13. Miller WL, Hartman KA, Burritt MF, et al. Serial biomarker measurements in ambulatory patients with chronic heart failure: the importance of change over time. Circulation 2007; 116(3):249–257. doi:10.1161/CIRCULATIONAHA.107.694562
  14. Logeart D, Beyne P, Cusson C, et al. Evidence of cardiac myolysis in severe nonischemic heart failure and the potential role of increased wall strain. Am Heart J 2001; 141(2):247–253. doi:10.1067/mhj.2001.111767
  15. Whittstein IS, Thiemann DR, Lima JA, et al. Neurohumoral features of myocardial stunning due to sudden emotional stress. N Engl J Med 2005; 352(6):539–548. doi:10.1056/NEJMoa043046
  16. McClennen S, Halamka JD, Horowitz GL, Kannam JP, Ho KK. Clinical prevalence and ramifications of false-positive cardiac troponin I elevations from the Abbott AxSYM Analyzer. Am J Cardiol 2003; 91(9):1125–1127.
  17. Bradham WS, Bian A, Oeser A, et al. High-sensitivity cardiac troponin-I is elevated in patients with rheumatoid arthritis, independent of cardiovascular risk factors and inflammation. PLoS One 2012; 7(6):e38930. doi:10.1371/journal.pone.0038930
  18. Li SF, Zapata J, Tillem E. The prevalence of false-positive cardiac troponin I in ED patients with rhabdomyolysis. Am J Emerg Med 2005; 23(7):860–863. doi:10.1016/j.ajem.2005.05.008
  19. Puelacher C, Twerenbold R, Mosimann T, et al. Effects of hemolysis on the diagnostic accuracy of cardiac troponin I for the diagnosis of myocardial infarction. Int J Cardiol 2015; 187:313–315. doi:10.1016/j.ijcard.2015.03.378
Issue
Cleveland Clinic Journal of Medicine - 85(4)
Issue
Cleveland Clinic Journal of Medicine - 85(4)
Page Number
274-277
Page Number
274-277
Publications
Publications
Topics
Article Type
Display Headline
Are serum troponin levels elevated in conditions other than acute coronary syndrome?
Display Headline
Are serum troponin levels elevated in conditions other than acute coronary syndrome?
Legacy Keywords
troponin, laboratory testing, acute coronary syndrome, myocardial infarction, MI, sepsis, stroke, chronic kidney disease, CKD, heart failure, stress cardiomyopathy, Kiran Sebastian, Alexander Wester, Anapama Kottam, Ayman Soubani
Legacy Keywords
troponin, laboratory testing, acute coronary syndrome, myocardial infarction, MI, sepsis, stroke, chronic kidney disease, CKD, heart failure, stress cardiomyopathy, Kiran Sebastian, Alexander Wester, Anapama Kottam, Ayman Soubani
Sections
Disallow All Ads
Content Gating
No Gating (article Unlocked/Free)
Alternative CME
Disqus Comments
Default
Gate On Date
Wed, 03/28/2018 - 11:30
Un-Gate On Date
Wed, 03/28/2018 - 11:30
Article PDF Media

Hypertension in older adults: What is the target blood pressure?

Article Type
Changed
Thu, 03/01/2018 - 08:55
Display Headline
Hypertension in older adults: What is the target blood pressure?

We should aim for a standard office systolic pressure lower than 130 mm Hg in most adults age 65 and older if the patient can take multiple antihypertensive medications and be followed closely for adverse effects.

This recommendation is part of the 2017 hypertension guideline from the American College of Cardiology and American Heart Association.1 This new guideline advocates drug treatment of hypertension to a target less than 130/80 mm Hg for patients of all ages for secondary prevention of cardiovascular disease, and for primary prevention in those at high risk (ie, an estimated 10-year risk of atherosclerotic cardiovascular disease of 10% or higher). The target blood pressure for those at lower risk is less than 140/90 mm Hg.

There are multiple tools to estimate the 10-year risk. All tools incorporate major predictors such as age, blood pressure, cholesterol profile, and other markers, depending on the tool. Although risk increases with age, the tools are inaccurate once the patient is approximately 80 years of age.

The recommendation for older adults omits a target diastolic pressure, since treating elevated systolic pressure has more data supporting it than treating elevated diastolic blood pressure in older people. These recommendations apply only to older adults who can walk and are living in the community, not in an institution, and includes the subset of older adults who have mild cognitive impairment and frailty. The goals of treatment should be patient-centered.

DATA BEHIND THE GUIDELINE: THE SPRINT TRIAL

The Systolic Blood Pressure Intervention Trial (SPRINT)2 enrolled 9,361 patients who, to enter, had to be at least 50 years old (the mean age was 67.9), have a systolic blood pressure of 130 to 180 mm Hg (the mean was 139.7 mm Hg), and be at risk of cardiovascular disease due to chronic kidney disease, clinical or subclinical cardiovascular disease, a 10-year Framingham risk score of at least 15%, or age 75 or older. They had few comorbidities, and patients with diabetes mellitus or prior stroke were excluded. The objective was to see if intensive blood pressure treatment reduced the incidence of adverse cardiovascular outcomes compared with standard control.

The participants were randomized to either an intensive treatment goal of systolic pressure less than 120 mm Hg or a standard treatment goal of less than 140 mm Hg. Investigators chose drugs and doses according to their clinical judgment. The study protocol called for blood pressure measurement using an untended automated cuff, which probably resulted in systolic pressure readings 5 to 10 mm Hg lower than with typical methods used in the office.3

The intensive treatment group achieved a mean systolic pressure of 121.5 mm Hg, which required an average of 3 drugs. In contrast, the standard treatment group achieved a systolic pressure of 136.2 mm Hg, which required an average of 1.9 drugs.

Due to an absolute risk reduction in cardiovascular events and mortality, SPRINT was discontinued early after a median follow-up of 3.3 years. In the entire cohort, 61 patients needed to be treated intensively to prevent 1 cardiovascular event, and 90 needed to be treated intensively to prevent 1 death.2

Favorable outcomes in the oldest subgroup

The oldest patients in the SPRINT trial tolerated the intensive treatment as well as the youngest.2,4

Exploratory analysis of the subgroup of patients age 75 and older, who constituted 28% of the patients in the trial, demonstrated significant benefit from intensive treatment. In this subgroup, 27 patients needed to be treated aggressively (compared with standard treatment) to prevent 1 cardiovascular event,  and 41 needed to be treated intensively to prevent 1 death.4 The lower numbers needing to be treated in the older subgroup than in the overall trial reflect the higher absolute risk in this older population.

Serious adverse events were more common with intensive treatment than with standard treatment in the subgroup of older patients who were frail.4 Emergency department visits or serious adverse events were more likely when gait speed (a measure of frailty) was missing from the medical record in the intensive treatment group compared with the standard treatment group. Hyponatremia (serum sodium level < 130 mmol/L) was more likely in the intensively treated group than in the standard treatment group. Although the rate of falls was higher in the oldest subgroup than in the overall SPRINT population, within this subgroup the rate of injurious falls resulting in an emergency department visit was lower with intensive treatment than with standard treatment (11.6% vs 14.1%, P = .04).4

Most of the oldest patients scored below the nominal cutoff for normal (26 points)5 on the 30-point Montreal Cognitive Assessment, and about one-quarter scored below 19, which may be consistent with a major neurocognitive disorder.6

The SPRINT investigators validated a frailty scale in the study patients and found that the most frail benefited from intensive blood pressure control, as did the slowest walkers.

SPRINT results do not apply to very frail, sick patients

For older patients with hypertension, a high burden of comorbidity, and a limited life expectancy, the 2017 guidelines defer treatment decisions to clinical judgment and patient preference.

There have been no randomized trials of blood pressure management for older adults with substantial comorbidities or dementia. The “frail” older adults in the SPRINT trial were still living in the community, without dementia. The intensively treated frail older adults had more serious adverse events than with standard treatment. Those who were documented as being unable to walk at the time of enrollment also had more serious adverse events. Institutionalized older adults and nonambulatory adults in the community would likely have even higher rates of serious adverse events with intensive treatment than the SPRINT patients, and there is concern for excessive adverse effects from intensive blood pressure control in more debilitated older patients.

 

 

DOES TREATING HIGH BLOOD PRESSURE PREVENT FRAILTY OR DEMENTIA?

Aging without frailty is an important goal of geriatric care and is likely related to cardiovascular health.7 An older adult who becomes slower physically or mentally, with diminished strength and energy, is less likely to be able to live independently.

Would treating systolic blood pressure to a target of 120 to 130 mm Hg reduce the risk of prefrailty or frailty? Unfortunately, the 3-year SPRINT follow-up of the adults age 75 and older did not show any effect of intensive treatment on gait speed or mobility limitation.8 It is possible that the early termination of the study limited outcomes.

Regarding cognition, the new guidelines say that lowering blood pressure in adults with hypertension to prevent cognitive decline and dementia is reasonable, giving it a class IIa (moderate) recommendation, but they do not offer a particular blood pressure target.

Two systematic reviews of randomized placebo-controlled trials9,10 suggested that pharmacologic treatment of hypertension reduces the progression of cognitive impairment. The trials did not use an intensive treatment goal.

The impact of intensive treatment of hypertension (to a target of 120–130 mm Hg) on the development or progression of cognitive impairment is not known at this time. The SPRINT Memory and Cognition in Decreased Hypertension analysis may shed light on the effect of intensive treatment of blood pressure on the incidence of dementia, although the early termination of SPRINT may limit its conclusions as well.

GOALS SHOULD BE PATIENT-CENTERED

The new hypertension guideline gives clinicians 2 things to think about when treating hypertensive, ambulatory, noninstitutionalized, nondemented older adults, including those age 75 and older:

  • Older adults tolerate intensive blood pressure treatment as well as standard treatment. In particular, the fall rate is not increased and may even be less with intensive treatment.
  • Older adults have better cardiovascular outcomes with blood pressure less than 130 mm Hg than with higher levels.

Adherence to the new guidelines would require many older adults without significant multimorbidity to take 3 drugs and undergo more frequent monitoring. This burden may align with the goals of care for many older adults. However, data do not exist to prove a benefit from intensive blood pressure control in debilitated elderly patients, and there may be harm. Lowering the medication burden may be a more important goal than lowering the pressure for this population. Blood pressure targets and hypertension management should reflect patient-centered goals of care.

References
  1. Whelton PK, Carey RM, Aronow WS, et al. 2017 ACC/AHA/AAPA/ABC/ACPM/AGS/APhA/ASH/ASPC/NMA/PCNA Guideline for the prevention, detection, evaluation, and management of high blood pressure in adults: executive summary: a report of the American College of Cardiology/American Heart Association Task Force on Clinical Practice Guidelines. Hypertension 2017. Epub ahead of print.
  2. SPRINT Research Group; Wright JT Jr, Williamson JD, et al. A randomized trial of intensive versus standard blood-pressure control. N Engl J Med 2015; 373:2103–2116.
  3. Bakris GL. The implications of blood pressure measurement methods on treatment targets for blood pressure. Circulation 2016; 134:904–905.
  4. Williamson JD, Supiano MA, Applegate WB, et al; SPRINT Research Group. Intensive vs standard blood pressure control and cardiovascular disease outcomes in adults aged ≥ 75 years: a randomized clinical trial. JAMA 2016; 315:2673–2682.
  5. Nasreddine ZS, Phillips NA, Bedirian V, et al. The Montreal Cognitive Assessment, MoCA: a brief screening tool for mild cognitive impairment. J Am Geriatr Soc 2005; 53:695–699.
  6. Borland E, Nagga K, Nilsson PM, Minthon L, Nilsson ED, Palmqvist S. The Montreal Cognitive Assessment: normative data from a large Swedish population-based cohort. J Alzheimers Dis 2017; 59:893–901.
  7. Graciani A, Garcia-Esquinas E, Lopez-Garcia E, Banegas JR, Rodriguez-Artalejo F. Ideal cardiovascular health and risk of frailty in older adults. Circ Cardiovasc Qual Outcomes 2016; 9:239–245.
  8. Odden MC, Peralta CA, Berlowitz DR, et al; Systolic Blood Pressure Intervention Trial (SPRINT) Research Group. Effect of intensive blood pressure control on gait speed and mobility limitation in adults 75 years or older: a randomized clinical trial. JAMA Intern Med 2017; 177:500–507.
  9. Tully PJ, Hanon O, Cosh S, Tzourio C. Diuretic antihypertensive drugs and incident dementia risk: a systematic review, meta-analysis and meta-regression of prospective studies. J Hypertens 2016; 34:1027–1035.
  10. Rouch L, Cestac P, Hanon O, et al. Antihypertensive drugs, prevention of cognitive decline and dementia: a systematic review of observational studies, randomized controlled trials and meta-analyses, with discussion of potential mechanisms. CNS Drugs 2015; 29:113–130.
Article PDF
Author and Disclosure Information

Anu Garg, MD
Department of Internal Medicine and Program Director, Geriatric Medicine Fellowship, University of Toledo Medical Center; Assistant Professor, University of Toledo College of Medicine and Life Sciences, Toledo, OH

Barbara J. Messinger-Rapport, MD, PhD
Department of Preventive Medicine, Cleveland Clinic; Associate Professor, Cleveland Clinic Lerner College of Medicine of Case Western Reserve University, Cleveland, OH

Address: Barbara J. Messinger-Rapport, MD, PhD, Department of Preventive Medicine, A11, Cleveland Clinic, 9500 Euclid Avenue, Cleveland, OH 44195; rapporb@ccf.org

Issue
Cleveland Clinic Journal of Medicine - 85(3)
Publications
Topics
Page Number
193-195
Legacy Keywords
hypertension, high blood pressure, goal, target, guidelines, SPRINT trial, older adults, Anu Garg, Barbara Messenger-Rapport
Sections
Author and Disclosure Information

Anu Garg, MD
Department of Internal Medicine and Program Director, Geriatric Medicine Fellowship, University of Toledo Medical Center; Assistant Professor, University of Toledo College of Medicine and Life Sciences, Toledo, OH

Barbara J. Messinger-Rapport, MD, PhD
Department of Preventive Medicine, Cleveland Clinic; Associate Professor, Cleveland Clinic Lerner College of Medicine of Case Western Reserve University, Cleveland, OH

Address: Barbara J. Messinger-Rapport, MD, PhD, Department of Preventive Medicine, A11, Cleveland Clinic, 9500 Euclid Avenue, Cleveland, OH 44195; rapporb@ccf.org

Author and Disclosure Information

Anu Garg, MD
Department of Internal Medicine and Program Director, Geriatric Medicine Fellowship, University of Toledo Medical Center; Assistant Professor, University of Toledo College of Medicine and Life Sciences, Toledo, OH

Barbara J. Messinger-Rapport, MD, PhD
Department of Preventive Medicine, Cleveland Clinic; Associate Professor, Cleveland Clinic Lerner College of Medicine of Case Western Reserve University, Cleveland, OH

Address: Barbara J. Messinger-Rapport, MD, PhD, Department of Preventive Medicine, A11, Cleveland Clinic, 9500 Euclid Avenue, Cleveland, OH 44195; rapporb@ccf.org

Article PDF
Article PDF
Related Articles

We should aim for a standard office systolic pressure lower than 130 mm Hg in most adults age 65 and older if the patient can take multiple antihypertensive medications and be followed closely for adverse effects.

This recommendation is part of the 2017 hypertension guideline from the American College of Cardiology and American Heart Association.1 This new guideline advocates drug treatment of hypertension to a target less than 130/80 mm Hg for patients of all ages for secondary prevention of cardiovascular disease, and for primary prevention in those at high risk (ie, an estimated 10-year risk of atherosclerotic cardiovascular disease of 10% or higher). The target blood pressure for those at lower risk is less than 140/90 mm Hg.

There are multiple tools to estimate the 10-year risk. All tools incorporate major predictors such as age, blood pressure, cholesterol profile, and other markers, depending on the tool. Although risk increases with age, the tools are inaccurate once the patient is approximately 80 years of age.

The recommendation for older adults omits a target diastolic pressure, since treating elevated systolic pressure has more data supporting it than treating elevated diastolic blood pressure in older people. These recommendations apply only to older adults who can walk and are living in the community, not in an institution, and includes the subset of older adults who have mild cognitive impairment and frailty. The goals of treatment should be patient-centered.

DATA BEHIND THE GUIDELINE: THE SPRINT TRIAL

The Systolic Blood Pressure Intervention Trial (SPRINT)2 enrolled 9,361 patients who, to enter, had to be at least 50 years old (the mean age was 67.9), have a systolic blood pressure of 130 to 180 mm Hg (the mean was 139.7 mm Hg), and be at risk of cardiovascular disease due to chronic kidney disease, clinical or subclinical cardiovascular disease, a 10-year Framingham risk score of at least 15%, or age 75 or older. They had few comorbidities, and patients with diabetes mellitus or prior stroke were excluded. The objective was to see if intensive blood pressure treatment reduced the incidence of adverse cardiovascular outcomes compared with standard control.

The participants were randomized to either an intensive treatment goal of systolic pressure less than 120 mm Hg or a standard treatment goal of less than 140 mm Hg. Investigators chose drugs and doses according to their clinical judgment. The study protocol called for blood pressure measurement using an untended automated cuff, which probably resulted in systolic pressure readings 5 to 10 mm Hg lower than with typical methods used in the office.3

The intensive treatment group achieved a mean systolic pressure of 121.5 mm Hg, which required an average of 3 drugs. In contrast, the standard treatment group achieved a systolic pressure of 136.2 mm Hg, which required an average of 1.9 drugs.

Due to an absolute risk reduction in cardiovascular events and mortality, SPRINT was discontinued early after a median follow-up of 3.3 years. In the entire cohort, 61 patients needed to be treated intensively to prevent 1 cardiovascular event, and 90 needed to be treated intensively to prevent 1 death.2

Favorable outcomes in the oldest subgroup

The oldest patients in the SPRINT trial tolerated the intensive treatment as well as the youngest.2,4

Exploratory analysis of the subgroup of patients age 75 and older, who constituted 28% of the patients in the trial, demonstrated significant benefit from intensive treatment. In this subgroup, 27 patients needed to be treated aggressively (compared with standard treatment) to prevent 1 cardiovascular event,  and 41 needed to be treated intensively to prevent 1 death.4 The lower numbers needing to be treated in the older subgroup than in the overall trial reflect the higher absolute risk in this older population.

Serious adverse events were more common with intensive treatment than with standard treatment in the subgroup of older patients who were frail.4 Emergency department visits or serious adverse events were more likely when gait speed (a measure of frailty) was missing from the medical record in the intensive treatment group compared with the standard treatment group. Hyponatremia (serum sodium level < 130 mmol/L) was more likely in the intensively treated group than in the standard treatment group. Although the rate of falls was higher in the oldest subgroup than in the overall SPRINT population, within this subgroup the rate of injurious falls resulting in an emergency department visit was lower with intensive treatment than with standard treatment (11.6% vs 14.1%, P = .04).4

Most of the oldest patients scored below the nominal cutoff for normal (26 points)5 on the 30-point Montreal Cognitive Assessment, and about one-quarter scored below 19, which may be consistent with a major neurocognitive disorder.6

The SPRINT investigators validated a frailty scale in the study patients and found that the most frail benefited from intensive blood pressure control, as did the slowest walkers.

SPRINT results do not apply to very frail, sick patients

For older patients with hypertension, a high burden of comorbidity, and a limited life expectancy, the 2017 guidelines defer treatment decisions to clinical judgment and patient preference.

There have been no randomized trials of blood pressure management for older adults with substantial comorbidities or dementia. The “frail” older adults in the SPRINT trial were still living in the community, without dementia. The intensively treated frail older adults had more serious adverse events than with standard treatment. Those who were documented as being unable to walk at the time of enrollment also had more serious adverse events. Institutionalized older adults and nonambulatory adults in the community would likely have even higher rates of serious adverse events with intensive treatment than the SPRINT patients, and there is concern for excessive adverse effects from intensive blood pressure control in more debilitated older patients.

 

 

DOES TREATING HIGH BLOOD PRESSURE PREVENT FRAILTY OR DEMENTIA?

Aging without frailty is an important goal of geriatric care and is likely related to cardiovascular health.7 An older adult who becomes slower physically or mentally, with diminished strength and energy, is less likely to be able to live independently.

Would treating systolic blood pressure to a target of 120 to 130 mm Hg reduce the risk of prefrailty or frailty? Unfortunately, the 3-year SPRINT follow-up of the adults age 75 and older did not show any effect of intensive treatment on gait speed or mobility limitation.8 It is possible that the early termination of the study limited outcomes.

Regarding cognition, the new guidelines say that lowering blood pressure in adults with hypertension to prevent cognitive decline and dementia is reasonable, giving it a class IIa (moderate) recommendation, but they do not offer a particular blood pressure target.

Two systematic reviews of randomized placebo-controlled trials9,10 suggested that pharmacologic treatment of hypertension reduces the progression of cognitive impairment. The trials did not use an intensive treatment goal.

The impact of intensive treatment of hypertension (to a target of 120–130 mm Hg) on the development or progression of cognitive impairment is not known at this time. The SPRINT Memory and Cognition in Decreased Hypertension analysis may shed light on the effect of intensive treatment of blood pressure on the incidence of dementia, although the early termination of SPRINT may limit its conclusions as well.

GOALS SHOULD BE PATIENT-CENTERED

The new hypertension guideline gives clinicians 2 things to think about when treating hypertensive, ambulatory, noninstitutionalized, nondemented older adults, including those age 75 and older:

  • Older adults tolerate intensive blood pressure treatment as well as standard treatment. In particular, the fall rate is not increased and may even be less with intensive treatment.
  • Older adults have better cardiovascular outcomes with blood pressure less than 130 mm Hg than with higher levels.

Adherence to the new guidelines would require many older adults without significant multimorbidity to take 3 drugs and undergo more frequent monitoring. This burden may align with the goals of care for many older adults. However, data do not exist to prove a benefit from intensive blood pressure control in debilitated elderly patients, and there may be harm. Lowering the medication burden may be a more important goal than lowering the pressure for this population. Blood pressure targets and hypertension management should reflect patient-centered goals of care.

We should aim for a standard office systolic pressure lower than 130 mm Hg in most adults age 65 and older if the patient can take multiple antihypertensive medications and be followed closely for adverse effects.

This recommendation is part of the 2017 hypertension guideline from the American College of Cardiology and American Heart Association.1 This new guideline advocates drug treatment of hypertension to a target less than 130/80 mm Hg for patients of all ages for secondary prevention of cardiovascular disease, and for primary prevention in those at high risk (ie, an estimated 10-year risk of atherosclerotic cardiovascular disease of 10% or higher). The target blood pressure for those at lower risk is less than 140/90 mm Hg.

There are multiple tools to estimate the 10-year risk. All tools incorporate major predictors such as age, blood pressure, cholesterol profile, and other markers, depending on the tool. Although risk increases with age, the tools are inaccurate once the patient is approximately 80 years of age.

The recommendation for older adults omits a target diastolic pressure, since treating elevated systolic pressure has more data supporting it than treating elevated diastolic blood pressure in older people. These recommendations apply only to older adults who can walk and are living in the community, not in an institution, and includes the subset of older adults who have mild cognitive impairment and frailty. The goals of treatment should be patient-centered.

DATA BEHIND THE GUIDELINE: THE SPRINT TRIAL

The Systolic Blood Pressure Intervention Trial (SPRINT)2 enrolled 9,361 patients who, to enter, had to be at least 50 years old (the mean age was 67.9), have a systolic blood pressure of 130 to 180 mm Hg (the mean was 139.7 mm Hg), and be at risk of cardiovascular disease due to chronic kidney disease, clinical or subclinical cardiovascular disease, a 10-year Framingham risk score of at least 15%, or age 75 or older. They had few comorbidities, and patients with diabetes mellitus or prior stroke were excluded. The objective was to see if intensive blood pressure treatment reduced the incidence of adverse cardiovascular outcomes compared with standard control.

The participants were randomized to either an intensive treatment goal of systolic pressure less than 120 mm Hg or a standard treatment goal of less than 140 mm Hg. Investigators chose drugs and doses according to their clinical judgment. The study protocol called for blood pressure measurement using an untended automated cuff, which probably resulted in systolic pressure readings 5 to 10 mm Hg lower than with typical methods used in the office.3

The intensive treatment group achieved a mean systolic pressure of 121.5 mm Hg, which required an average of 3 drugs. In contrast, the standard treatment group achieved a systolic pressure of 136.2 mm Hg, which required an average of 1.9 drugs.

Due to an absolute risk reduction in cardiovascular events and mortality, SPRINT was discontinued early after a median follow-up of 3.3 years. In the entire cohort, 61 patients needed to be treated intensively to prevent 1 cardiovascular event, and 90 needed to be treated intensively to prevent 1 death.2

Favorable outcomes in the oldest subgroup

The oldest patients in the SPRINT trial tolerated the intensive treatment as well as the youngest.2,4

Exploratory analysis of the subgroup of patients age 75 and older, who constituted 28% of the patients in the trial, demonstrated significant benefit from intensive treatment. In this subgroup, 27 patients needed to be treated aggressively (compared with standard treatment) to prevent 1 cardiovascular event,  and 41 needed to be treated intensively to prevent 1 death.4 The lower numbers needing to be treated in the older subgroup than in the overall trial reflect the higher absolute risk in this older population.

Serious adverse events were more common with intensive treatment than with standard treatment in the subgroup of older patients who were frail.4 Emergency department visits or serious adverse events were more likely when gait speed (a measure of frailty) was missing from the medical record in the intensive treatment group compared with the standard treatment group. Hyponatremia (serum sodium level < 130 mmol/L) was more likely in the intensively treated group than in the standard treatment group. Although the rate of falls was higher in the oldest subgroup than in the overall SPRINT population, within this subgroup the rate of injurious falls resulting in an emergency department visit was lower with intensive treatment than with standard treatment (11.6% vs 14.1%, P = .04).4

Most of the oldest patients scored below the nominal cutoff for normal (26 points)5 on the 30-point Montreal Cognitive Assessment, and about one-quarter scored below 19, which may be consistent with a major neurocognitive disorder.6

The SPRINT investigators validated a frailty scale in the study patients and found that the most frail benefited from intensive blood pressure control, as did the slowest walkers.

SPRINT results do not apply to very frail, sick patients

For older patients with hypertension, a high burden of comorbidity, and a limited life expectancy, the 2017 guidelines defer treatment decisions to clinical judgment and patient preference.

There have been no randomized trials of blood pressure management for older adults with substantial comorbidities or dementia. The “frail” older adults in the SPRINT trial were still living in the community, without dementia. The intensively treated frail older adults had more serious adverse events than with standard treatment. Those who were documented as being unable to walk at the time of enrollment also had more serious adverse events. Institutionalized older adults and nonambulatory adults in the community would likely have even higher rates of serious adverse events with intensive treatment than the SPRINT patients, and there is concern for excessive adverse effects from intensive blood pressure control in more debilitated older patients.

 

 

DOES TREATING HIGH BLOOD PRESSURE PREVENT FRAILTY OR DEMENTIA?

Aging without frailty is an important goal of geriatric care and is likely related to cardiovascular health.7 An older adult who becomes slower physically or mentally, with diminished strength and energy, is less likely to be able to live independently.

Would treating systolic blood pressure to a target of 120 to 130 mm Hg reduce the risk of prefrailty or frailty? Unfortunately, the 3-year SPRINT follow-up of the adults age 75 and older did not show any effect of intensive treatment on gait speed or mobility limitation.8 It is possible that the early termination of the study limited outcomes.

Regarding cognition, the new guidelines say that lowering blood pressure in adults with hypertension to prevent cognitive decline and dementia is reasonable, giving it a class IIa (moderate) recommendation, but they do not offer a particular blood pressure target.

Two systematic reviews of randomized placebo-controlled trials9,10 suggested that pharmacologic treatment of hypertension reduces the progression of cognitive impairment. The trials did not use an intensive treatment goal.

The impact of intensive treatment of hypertension (to a target of 120–130 mm Hg) on the development or progression of cognitive impairment is not known at this time. The SPRINT Memory and Cognition in Decreased Hypertension analysis may shed light on the effect of intensive treatment of blood pressure on the incidence of dementia, although the early termination of SPRINT may limit its conclusions as well.

GOALS SHOULD BE PATIENT-CENTERED

The new hypertension guideline gives clinicians 2 things to think about when treating hypertensive, ambulatory, noninstitutionalized, nondemented older adults, including those age 75 and older:

  • Older adults tolerate intensive blood pressure treatment as well as standard treatment. In particular, the fall rate is not increased and may even be less with intensive treatment.
  • Older adults have better cardiovascular outcomes with blood pressure less than 130 mm Hg than with higher levels.

Adherence to the new guidelines would require many older adults without significant multimorbidity to take 3 drugs and undergo more frequent monitoring. This burden may align with the goals of care for many older adults. However, data do not exist to prove a benefit from intensive blood pressure control in debilitated elderly patients, and there may be harm. Lowering the medication burden may be a more important goal than lowering the pressure for this population. Blood pressure targets and hypertension management should reflect patient-centered goals of care.

References
  1. Whelton PK, Carey RM, Aronow WS, et al. 2017 ACC/AHA/AAPA/ABC/ACPM/AGS/APhA/ASH/ASPC/NMA/PCNA Guideline for the prevention, detection, evaluation, and management of high blood pressure in adults: executive summary: a report of the American College of Cardiology/American Heart Association Task Force on Clinical Practice Guidelines. Hypertension 2017. Epub ahead of print.
  2. SPRINT Research Group; Wright JT Jr, Williamson JD, et al. A randomized trial of intensive versus standard blood-pressure control. N Engl J Med 2015; 373:2103–2116.
  3. Bakris GL. The implications of blood pressure measurement methods on treatment targets for blood pressure. Circulation 2016; 134:904–905.
  4. Williamson JD, Supiano MA, Applegate WB, et al; SPRINT Research Group. Intensive vs standard blood pressure control and cardiovascular disease outcomes in adults aged ≥ 75 years: a randomized clinical trial. JAMA 2016; 315:2673–2682.
  5. Nasreddine ZS, Phillips NA, Bedirian V, et al. The Montreal Cognitive Assessment, MoCA: a brief screening tool for mild cognitive impairment. J Am Geriatr Soc 2005; 53:695–699.
  6. Borland E, Nagga K, Nilsson PM, Minthon L, Nilsson ED, Palmqvist S. The Montreal Cognitive Assessment: normative data from a large Swedish population-based cohort. J Alzheimers Dis 2017; 59:893–901.
  7. Graciani A, Garcia-Esquinas E, Lopez-Garcia E, Banegas JR, Rodriguez-Artalejo F. Ideal cardiovascular health and risk of frailty in older adults. Circ Cardiovasc Qual Outcomes 2016; 9:239–245.
  8. Odden MC, Peralta CA, Berlowitz DR, et al; Systolic Blood Pressure Intervention Trial (SPRINT) Research Group. Effect of intensive blood pressure control on gait speed and mobility limitation in adults 75 years or older: a randomized clinical trial. JAMA Intern Med 2017; 177:500–507.
  9. Tully PJ, Hanon O, Cosh S, Tzourio C. Diuretic antihypertensive drugs and incident dementia risk: a systematic review, meta-analysis and meta-regression of prospective studies. J Hypertens 2016; 34:1027–1035.
  10. Rouch L, Cestac P, Hanon O, et al. Antihypertensive drugs, prevention of cognitive decline and dementia: a systematic review of observational studies, randomized controlled trials and meta-analyses, with discussion of potential mechanisms. CNS Drugs 2015; 29:113–130.
References
  1. Whelton PK, Carey RM, Aronow WS, et al. 2017 ACC/AHA/AAPA/ABC/ACPM/AGS/APhA/ASH/ASPC/NMA/PCNA Guideline for the prevention, detection, evaluation, and management of high blood pressure in adults: executive summary: a report of the American College of Cardiology/American Heart Association Task Force on Clinical Practice Guidelines. Hypertension 2017. Epub ahead of print.
  2. SPRINT Research Group; Wright JT Jr, Williamson JD, et al. A randomized trial of intensive versus standard blood-pressure control. N Engl J Med 2015; 373:2103–2116.
  3. Bakris GL. The implications of blood pressure measurement methods on treatment targets for blood pressure. Circulation 2016; 134:904–905.
  4. Williamson JD, Supiano MA, Applegate WB, et al; SPRINT Research Group. Intensive vs standard blood pressure control and cardiovascular disease outcomes in adults aged ≥ 75 years: a randomized clinical trial. JAMA 2016; 315:2673–2682.
  5. Nasreddine ZS, Phillips NA, Bedirian V, et al. The Montreal Cognitive Assessment, MoCA: a brief screening tool for mild cognitive impairment. J Am Geriatr Soc 2005; 53:695–699.
  6. Borland E, Nagga K, Nilsson PM, Minthon L, Nilsson ED, Palmqvist S. The Montreal Cognitive Assessment: normative data from a large Swedish population-based cohort. J Alzheimers Dis 2017; 59:893–901.
  7. Graciani A, Garcia-Esquinas E, Lopez-Garcia E, Banegas JR, Rodriguez-Artalejo F. Ideal cardiovascular health and risk of frailty in older adults. Circ Cardiovasc Qual Outcomes 2016; 9:239–245.
  8. Odden MC, Peralta CA, Berlowitz DR, et al; Systolic Blood Pressure Intervention Trial (SPRINT) Research Group. Effect of intensive blood pressure control on gait speed and mobility limitation in adults 75 years or older: a randomized clinical trial. JAMA Intern Med 2017; 177:500–507.
  9. Tully PJ, Hanon O, Cosh S, Tzourio C. Diuretic antihypertensive drugs and incident dementia risk: a systematic review, meta-analysis and meta-regression of prospective studies. J Hypertens 2016; 34:1027–1035.
  10. Rouch L, Cestac P, Hanon O, et al. Antihypertensive drugs, prevention of cognitive decline and dementia: a systematic review of observational studies, randomized controlled trials and meta-analyses, with discussion of potential mechanisms. CNS Drugs 2015; 29:113–130.
Issue
Cleveland Clinic Journal of Medicine - 85(3)
Issue
Cleveland Clinic Journal of Medicine - 85(3)
Page Number
193-195
Page Number
193-195
Publications
Publications
Topics
Article Type
Display Headline
Hypertension in older adults: What is the target blood pressure?
Display Headline
Hypertension in older adults: What is the target blood pressure?
Legacy Keywords
hypertension, high blood pressure, goal, target, guidelines, SPRINT trial, older adults, Anu Garg, Barbara Messenger-Rapport
Legacy Keywords
hypertension, high blood pressure, goal, target, guidelines, SPRINT trial, older adults, Anu Garg, Barbara Messenger-Rapport
Sections
Disallow All Ads
Content Gating
No Gating (article Unlocked/Free)
Alternative CME
Disqus Comments
Default
Gate On Date
Fri, 02/23/2018 - 11:30
Un-Gate On Date
Fri, 02/23/2018 - 11:30
Article PDF Media

Do cardiac risk stratification indexes accurately estimate perioperative risk in noncardiac surgery patients?

Article Type
Changed
Mon, 01/08/2018 - 09:11
Display Headline
Do cardiac risk stratification indexes accurately estimate perioperative risk in noncardiac surgery patients?

Neither of the two cardiac risk assessment indexes most commonly used (Table 1)1,2 is completely accurate, nor is one superior to the other. To provide the most accurate assessment of cardiac risk, practitioners need to select the index most applicable to the circumstances of the individual patient.

CARDIAC COMPLICATIONS ARE INCREASING

The Revised Cardiac Risk Index and the National Surgical Quality Improvement Program index
About 5% of patients undergoing noncardiac surgery have a major cardiac complication within the first 30 postoperative days.3,4 This rate has been rising, primarily due to an increasing prevalence of cardiac comorbidities. Thus, accurate preoperative cardiac risk stratification is needed to assess the risk of perioperative major cardiac complications in all patients scheduled for noncardiac surgery. This information helps the perioperative team and patient to better weigh the benefits and risks of surgery and to optimize its timing and location (eg, inpatient vs outpatient surgery center).

CARDIAC RISK ASSESSMENT INDEXES

The 2 risk assessment indexes most often used are:

  • The Revised Cardiac Risk Index (RCRI)1
  • The National Surgical Quality Improvement Program (NSQIP) risk index, also known as the Gupta index.2

Both are endorsed by the American College of Cardiology (ACC) and the American Heart Association (AHA).5 The RCRI, introduced in 1999, is more commonly used, but the NSQIP, introduced in 2011, is based on a larger sample size.

Both indexes consider various factors in estimating the risk, with some overlap. The main outcome assessed in both indexes is the risk of a major cardiac event, ie, myocardial infarction or cardiac arrest. The RCRI outcome also includes ventricular fibrillation, complete heart block, and pulmonary edema, which may be sequelae to cardiac arrest and myocardial infarction. This difference in defined outcomes between the indexes is not likely to account for a significant variation in the prediction of risk; however, this is difficult to prove.

Each index defines myocardial infarction differently. The current clinical definition6 includes detection of a rise or fall of cardiac biomarker values (preferably cardiac troponins) with at least 1 value above the 99th percentile upper reference limit and at least 1 of the following:

  • Symptoms of ischemia
  • New ST-T wave changes or new left bundle branch block
  • New pathologic Q waves
  • Imaging evidence of new loss of viable myocardium tissue or new regional wall- motion abnormality
  • Finding of an intracoronary thrombus.

As seen in Table 1, the definition of myocardial infarction in NSQIP was one of the following: ST-segment elevation, new left bundle branch block, Q waves, or a troponin level greater than 3 times normal. Patients may have mild troponin leak of unknown significance without chest pain after surgery. This suggests that NSQIP may have overdiagnosed myocardial infarction.

USE IN CLINICAL PRACTICE

In clinical practice, which risk index is more accurate? Should clinicians become familiar with one index and keep using it? The 2014 ACC/AHA guidelines5 do not recommend one over the other, nor do they define the clinical situations that could lead to significant underestimation of risk.

The following are cases in which the indexes provide contradictory risk assessments.

Case 1. A 60-year-old man scheduled for surgery has diabetes mellitus, for which he takes insulin, and stable heart failure (left ventricular ejection fraction 40%). His RCRI score is 2, indicating an elevated 7% risk of cardiac complications; however, his NSQIP index is 0.31%. In this case, the NSQIP index probably underestimates the risk, as insulin-dependent diabetes and heart failure are not variables in the NSQIP index.

Case 2. A 60-year-old man who is partially functionally dependent and is on oxygen for severe chronic obstructive pulmonary disease is scheduled for craniotomy. His RCRI score is 0 (low risk), but his NSQIP index score (4.87%) indicates an elevated risk of cardiac complications based on his functional status, symptomatic chronic obstructive pulmonary disease, and high-risk surgery. In this case, the RCRI probably underestimates the risk.

These cases show that practitioners should not rely on just one index, but should rather decide which index to apply case by case. This avoids underestimating the risk. In patients with poor functional status and higher American Society of Anesthesiology class, the NSQIP index may provide a more accurate risk estimation than the RCRI. Patients with cardiomyopathy as well as those with insulin-dependent diabetes may be well assessed by the RCRI.

The following situations require additional caution when using these indexes, to avoid over- and underestimating cardiac risk.

 

 

PATIENTS WITH SEVERE AORTIC STENOSIS

Neither index lists severe aortic stenosis as a risk factor. The RCRI derivation and validation studies had only 5 patients with severe aortic stenosis, and the NSQIP validation study did not include any patients with aortic stenosis. Nevertheless, severe aortic stenosis increases the risk of cardiac complications in the perioperative period,7 making it important to consider in these patients.

Although patients with severe symptomatic aortic stenosis need valvular intervention before the surgery, patients who have asymptomatic severe aortic stenosis without associated cardiac dysfunction do not. Close hemodynamic monitoring during surgery is reasonable in the latter group.5,7

PATIENTS WITH RECENT STROKE

What would be the cardiac risk for a patient scheduled for elective hip surgery who has had a stroke within the last 3 months? If one applies both indexes, the cardiac risk comes to less than 1% (low risk) in both cases. However, this could be deceiving. A large study8 published in 2014 showed an elevated risk of cardiac complications in patients undergoing noncardiac surgery who had had an ischemic stroke within the previous 6 months; in the first 3 months, the odds ratio of developing a major adverse cardiovascular event was 14.23.This clearly overrides the traditional expert opinion-based evidence, which is that a time lapse of only 1 month after an ischemic stroke is safe for surgery.

PATIENTS WITH DIASTOLIC DYSFUNCTION

A 2016 meta-analysis and systematic review found that preoperative diastolic dysfunction was associated with higher rates of postoperative mortality and major adverse cardiac events, regardless of the left ventricular ejection fraction.9 However, the studies investigated included mostly patients undergoing cardiovascular surgeries. This raises the question of whether asymptomatic patients need echocardiography before surgery.

In a patient who has diastolic dysfunction, one should maintain adequate blood pressure control and euvolemia before the surgery and avoid hypertensive spikes in the immediate perioperative period, as hypertension is the worst enemy of those with diastolic dysfunction. Patients with atrial fibrillation may need more stringent heart rate control.

In a prospective study involving 1,005 consecutive vascular surgery patients, the 30-day cardiovascular event rate was highest in patients with symptomatic heart failure (49%), followed by those with asymptomatic systolic left ventricular dysfunction (23%), asymptomatic diastolic left ventricular dysfunction (18%), and normal left ventricular function (10%).10

Further studies are needed to determine whether the data obtained from the assessment of ventricular function in patients without signs or symptoms are significant enough to require updates to the criteria.

WHAT ABOUT THE ROLE OF BNP?

In a meta-analysis of 15 noncardiac surgery studies in 850 patients, preoperative B-type natriuretic peptide (BNP) levels independently predicted major adverse cardiac events, with levels greater than 372 pg/mL having a 36.7% incidence of major adverse cardiac events.11

A recent publication by the Canadian Cardiovascular Society12 strongly recommended measuring N-terminal-proBNP or BNP before noncardiac surgery to enhance perioperative cardiac risk estimation in patients who are age 65 or older, patients who are age 45 to 64 with significant cardiovascular disease, or patients who have an RCRI score of 1 or higher.

Further prospective randomized studies are needed to assess the utility of measuring BNP for preoperative cardiac risk evaluation.

PATIENTS WITH OBSTRUCTIVE SLEEP APNEA

Patients with obstructive sleep apnea scheduled for surgery under anesthesia have a higher risk of perioperative complications than patients without the disease, including higher rates of cardiac complications and atrial fibrillation. However, the evidence is insufficient to support canceling or delaying surgery in patients with suspected obstructive sleep apnea.

After comorbid conditions are optimally treated, patients with obstructive sleep apnea can proceed to surgery, provided strategies for mitigating complications are implemented.13

 

 

TO STRESS OR NOT TO STRESS?

A common question is whether to perform a stress test before surgery. Based on the ACC/AHA guidelines,5 preoperative stress testing is not indicated solely to assess surgical risk if there is no other indication for it.

Stress testing can be used to determine whether the patient needs coronary revascularization. However, routine coronary revascularization is not recommended before noncardiac surgery exclusively to reduce perioperative cardiac events.

This conclusion is based on a landmark trial in which revascularization had no significant effect on outcomes.14 That trial included high-risk patients undergoing major vascular surgery who had greater than 70% stenosis of 1 or more major coronary arteries on angiography, randomized to either revascularization or no revascularization. It excluded patients with severe left main artery disease, ejection fraction less than 20%, and severe aortic stenosis. Results showed no differences in the rates of postoperative death, myocardial infarction, and stroke between the 2 groups. Furthermore, there was no postoperative survival difference during 5 years of follow-up.

Stress testing may be considered for patients with elevated risk and whose functional capacity is poor (< 4 metabolic equivalents) or unknown if it will change the management strategy. Another consideration affecting whether to perform stress testing is whether the surgery can be deferred for a month if the stress test is positive and a bare-metal coronary stent is placed, to allow for completion of dual antiplatelet therapy.

SHOULD WE ROUTINELY MONITOR TROPONIN AFTER SURGERY IN ASYMPTOMATIC PATIENTS?

Currently, the role of routine monitoring of troponin postoperatively in asymptomatic patients is unclear. The Canadian Cardiovascular Society12 recommends monitoring troponin in selected group of patients, eg, those with an RCRI score of 1 or higher, age 65 or older, a significant cardiac history, or elevated BNP preoperatively. However, at this point we do not have strong evidence regarding the implications of mild asymptomatic troponin elevation postoperatively and how to manage it. Two currently ongoing randomized controlled trials will answer those questions:

  • The Management of Myocardial Injury After Noncardiac Surgery (MANAGE) trial, comparing the use of dabigatran and omeprazole vs placebo in myocardial injury postoperatively
  • The Study of Ticagrelor Versus Aspirin Treatment in Patients With Myocardial Injury Post Major Non-cardiac Surgery (INTREPID).
References
  1. Lee TH, Marcantonio ER, Mangione CM, et al. Derivation and prospective validation of a simple index for prediction of cardiac risk of major noncardiac surgery. Circulation 1999; 100:1043–1049.
  2. Gupta PK, Gupta H, Sundaram A, et al. Development and validation of a risk calculator for prediction of cardiac risk after surgery. Circulation 2011; 124:381–387.
  3. Devereaux PJ, Sessler DI. Cardiac complications in patients undergoing major noncardiac surgery. N Engl J Med 2015; 373:2258–2269.
  4. Smilowitz NR, Gupta N, Ramakrishna H, Guo Y, Berger JS, Bangalore S. Perioperative major adverse cardiovascular and cerebrovascular events associated with noncardiac surgery. JAMA Cardiol 2017; 2:181–187.
  5. Fleisher LA, Fleischmann KE, Auerbach AD, et al; American College of Cardiology; American Heart Association. 2014 ACC/AHA guideline on perioperative cardiovascular evaluation and management of patients undergoing noncardiac surgery: a report of the American College of Cardiology/American Heart Association Task Force on practice guidelines. J Am Coll Cardiol 2014; 64:e77–e137 [Simultaneous publication: Circulation 2014; 130:2215–2245].
  6. Thygesen K, Alpert JS, Jaffe AS, et al, for the Joint ESC/ACCF/AHA/WHF Task Force for the Universal Definition of Myocardial Infarction. Third universal definition of myocardial infarction. Circulation 2012; 126:2020–2035.
  7. Tashiro T, Pislaru SV, Blustin JM, et al. Perioperative risk of major non-cardiac surgery in patients with severe aortic stenosis: a reappraisal in contemporary practice. Eur Heart J 2014; 35:2372–2381.
  8. Jørgensen ME, Torp-Pedersen C, Gislason GH, et al. Time elapsed after ischemic stroke and risk of adverse cardiovascular events and mortality following elective noncardiac surgery. JAMA 2014; 312:269–277.
  9. Kaw R, Hernandez AV, Pasupuleti V, et al; Cardiovascular Meta-analyses Research Group. Effect of diastolic dysfunction on postoperative outcomes after cardiovascular surgery: a systematic review and meta-analysis. J Thorac Cardiovasc Surg 2016; 152:1142–1153.
  10. Flu WJ, van Kuijk JP, Hoeks SE, et al. Prognostic implications of asymptomatic left ventricular dysfunction in patients undergoing vascular surgery. Anesthesiology 2010; 112:1316–1324.
  11. Rodseth R, Lurati Buse G, Bolliger D, et al. The predictive ability of pre-operative B-type natriuretic peptide in vascular patients for major adverse cardiac events: an individual patient data meta-analysis. J Am Coll Cardiol 2011; 58:522–529.
  12. Duceppe E, Parlow J, MacDonald P, et al. Canadian Cardiovascular Society Guidelines on perioperative cardiac risk assessment and management for patients who undergo noncardiac surgery. Can J Cardiol 2017; 33:17–32.
  13. Chung F, Memtsoudis SG, Ramachandran SK, et al. Society of Anesthesia and Sleep Medicine guidelines on preoperative screening and assessment of adult patients with obstructive sleep apnea. Anesth Analg 2016; 123:452–473.
  14. McFalls EO, Ward HB, Moritz TE, et al. Coronary-artery revascularization before elective major vascular surgery. N Engl J Med 2004; 351:2795–2804.
Article PDF
Author and Disclosure Information

Rohan Mandaliya, MD, FACP
Clinical Fellow, Division of Gastroenterology and Hepatology, Department of Medicine, Georgetown University Hospital, Washington, DC

Geno Merli, MD, MACP
Professor and Co-director, Jefferson Vascular Center, Department of Medicine, Thomas Jefferson University Hospital, Philadelphia, PA

Address: Rohan Mandaliya, MD, FACP, Department of Medicine, Georgetown University Hospital, 3800 Reservoir Road NW, Washington, DC, 20007; rohan86m@gmail.com

Issue
Cleveland Clinic Journal of Medicine - 85(1)
Publications
Topics
Page Number
35-39
Legacy Keywords
risk stratification, perioperative, cardiac, risk indexes, perioperative, surgery, Revised Cardiac Risk Index, RCRI, National Surgical Quality Improvement Program, NSQIP, myocardial infarction, MI, Rohan Mandaliya, Geno Merli
Sections
Author and Disclosure Information

Rohan Mandaliya, MD, FACP
Clinical Fellow, Division of Gastroenterology and Hepatology, Department of Medicine, Georgetown University Hospital, Washington, DC

Geno Merli, MD, MACP
Professor and Co-director, Jefferson Vascular Center, Department of Medicine, Thomas Jefferson University Hospital, Philadelphia, PA

Address: Rohan Mandaliya, MD, FACP, Department of Medicine, Georgetown University Hospital, 3800 Reservoir Road NW, Washington, DC, 20007; rohan86m@gmail.com

Author and Disclosure Information

Rohan Mandaliya, MD, FACP
Clinical Fellow, Division of Gastroenterology and Hepatology, Department of Medicine, Georgetown University Hospital, Washington, DC

Geno Merli, MD, MACP
Professor and Co-director, Jefferson Vascular Center, Department of Medicine, Thomas Jefferson University Hospital, Philadelphia, PA

Address: Rohan Mandaliya, MD, FACP, Department of Medicine, Georgetown University Hospital, 3800 Reservoir Road NW, Washington, DC, 20007; rohan86m@gmail.com

Article PDF
Article PDF

Neither of the two cardiac risk assessment indexes most commonly used (Table 1)1,2 is completely accurate, nor is one superior to the other. To provide the most accurate assessment of cardiac risk, practitioners need to select the index most applicable to the circumstances of the individual patient.

CARDIAC COMPLICATIONS ARE INCREASING

The Revised Cardiac Risk Index and the National Surgical Quality Improvement Program index
About 5% of patients undergoing noncardiac surgery have a major cardiac complication within the first 30 postoperative days.3,4 This rate has been rising, primarily due to an increasing prevalence of cardiac comorbidities. Thus, accurate preoperative cardiac risk stratification is needed to assess the risk of perioperative major cardiac complications in all patients scheduled for noncardiac surgery. This information helps the perioperative team and patient to better weigh the benefits and risks of surgery and to optimize its timing and location (eg, inpatient vs outpatient surgery center).

CARDIAC RISK ASSESSMENT INDEXES

The 2 risk assessment indexes most often used are:

  • The Revised Cardiac Risk Index (RCRI)1
  • The National Surgical Quality Improvement Program (NSQIP) risk index, also known as the Gupta index.2

Both are endorsed by the American College of Cardiology (ACC) and the American Heart Association (AHA).5 The RCRI, introduced in 1999, is more commonly used, but the NSQIP, introduced in 2011, is based on a larger sample size.

Both indexes consider various factors in estimating the risk, with some overlap. The main outcome assessed in both indexes is the risk of a major cardiac event, ie, myocardial infarction or cardiac arrest. The RCRI outcome also includes ventricular fibrillation, complete heart block, and pulmonary edema, which may be sequelae to cardiac arrest and myocardial infarction. This difference in defined outcomes between the indexes is not likely to account for a significant variation in the prediction of risk; however, this is difficult to prove.

Each index defines myocardial infarction differently. The current clinical definition6 includes detection of a rise or fall of cardiac biomarker values (preferably cardiac troponins) with at least 1 value above the 99th percentile upper reference limit and at least 1 of the following:

  • Symptoms of ischemia
  • New ST-T wave changes or new left bundle branch block
  • New pathologic Q waves
  • Imaging evidence of new loss of viable myocardium tissue or new regional wall- motion abnormality
  • Finding of an intracoronary thrombus.

As seen in Table 1, the definition of myocardial infarction in NSQIP was one of the following: ST-segment elevation, new left bundle branch block, Q waves, or a troponin level greater than 3 times normal. Patients may have mild troponin leak of unknown significance without chest pain after surgery. This suggests that NSQIP may have overdiagnosed myocardial infarction.

USE IN CLINICAL PRACTICE

In clinical practice, which risk index is more accurate? Should clinicians become familiar with one index and keep using it? The 2014 ACC/AHA guidelines5 do not recommend one over the other, nor do they define the clinical situations that could lead to significant underestimation of risk.

The following are cases in which the indexes provide contradictory risk assessments.

Case 1. A 60-year-old man scheduled for surgery has diabetes mellitus, for which he takes insulin, and stable heart failure (left ventricular ejection fraction 40%). His RCRI score is 2, indicating an elevated 7% risk of cardiac complications; however, his NSQIP index is 0.31%. In this case, the NSQIP index probably underestimates the risk, as insulin-dependent diabetes and heart failure are not variables in the NSQIP index.

Case 2. A 60-year-old man who is partially functionally dependent and is on oxygen for severe chronic obstructive pulmonary disease is scheduled for craniotomy. His RCRI score is 0 (low risk), but his NSQIP index score (4.87%) indicates an elevated risk of cardiac complications based on his functional status, symptomatic chronic obstructive pulmonary disease, and high-risk surgery. In this case, the RCRI probably underestimates the risk.

These cases show that practitioners should not rely on just one index, but should rather decide which index to apply case by case. This avoids underestimating the risk. In patients with poor functional status and higher American Society of Anesthesiology class, the NSQIP index may provide a more accurate risk estimation than the RCRI. Patients with cardiomyopathy as well as those with insulin-dependent diabetes may be well assessed by the RCRI.

The following situations require additional caution when using these indexes, to avoid over- and underestimating cardiac risk.

 

 

PATIENTS WITH SEVERE AORTIC STENOSIS

Neither index lists severe aortic stenosis as a risk factor. The RCRI derivation and validation studies had only 5 patients with severe aortic stenosis, and the NSQIP validation study did not include any patients with aortic stenosis. Nevertheless, severe aortic stenosis increases the risk of cardiac complications in the perioperative period,7 making it important to consider in these patients.

Although patients with severe symptomatic aortic stenosis need valvular intervention before the surgery, patients who have asymptomatic severe aortic stenosis without associated cardiac dysfunction do not. Close hemodynamic monitoring during surgery is reasonable in the latter group.5,7

PATIENTS WITH RECENT STROKE

What would be the cardiac risk for a patient scheduled for elective hip surgery who has had a stroke within the last 3 months? If one applies both indexes, the cardiac risk comes to less than 1% (low risk) in both cases. However, this could be deceiving. A large study8 published in 2014 showed an elevated risk of cardiac complications in patients undergoing noncardiac surgery who had had an ischemic stroke within the previous 6 months; in the first 3 months, the odds ratio of developing a major adverse cardiovascular event was 14.23.This clearly overrides the traditional expert opinion-based evidence, which is that a time lapse of only 1 month after an ischemic stroke is safe for surgery.

PATIENTS WITH DIASTOLIC DYSFUNCTION

A 2016 meta-analysis and systematic review found that preoperative diastolic dysfunction was associated with higher rates of postoperative mortality and major adverse cardiac events, regardless of the left ventricular ejection fraction.9 However, the studies investigated included mostly patients undergoing cardiovascular surgeries. This raises the question of whether asymptomatic patients need echocardiography before surgery.

In a patient who has diastolic dysfunction, one should maintain adequate blood pressure control and euvolemia before the surgery and avoid hypertensive spikes in the immediate perioperative period, as hypertension is the worst enemy of those with diastolic dysfunction. Patients with atrial fibrillation may need more stringent heart rate control.

In a prospective study involving 1,005 consecutive vascular surgery patients, the 30-day cardiovascular event rate was highest in patients with symptomatic heart failure (49%), followed by those with asymptomatic systolic left ventricular dysfunction (23%), asymptomatic diastolic left ventricular dysfunction (18%), and normal left ventricular function (10%).10

Further studies are needed to determine whether the data obtained from the assessment of ventricular function in patients without signs or symptoms are significant enough to require updates to the criteria.

WHAT ABOUT THE ROLE OF BNP?

In a meta-analysis of 15 noncardiac surgery studies in 850 patients, preoperative B-type natriuretic peptide (BNP) levels independently predicted major adverse cardiac events, with levels greater than 372 pg/mL having a 36.7% incidence of major adverse cardiac events.11

A recent publication by the Canadian Cardiovascular Society12 strongly recommended measuring N-terminal-proBNP or BNP before noncardiac surgery to enhance perioperative cardiac risk estimation in patients who are age 65 or older, patients who are age 45 to 64 with significant cardiovascular disease, or patients who have an RCRI score of 1 or higher.

Further prospective randomized studies are needed to assess the utility of measuring BNP for preoperative cardiac risk evaluation.

PATIENTS WITH OBSTRUCTIVE SLEEP APNEA

Patients with obstructive sleep apnea scheduled for surgery under anesthesia have a higher risk of perioperative complications than patients without the disease, including higher rates of cardiac complications and atrial fibrillation. However, the evidence is insufficient to support canceling or delaying surgery in patients with suspected obstructive sleep apnea.

After comorbid conditions are optimally treated, patients with obstructive sleep apnea can proceed to surgery, provided strategies for mitigating complications are implemented.13

 

 

TO STRESS OR NOT TO STRESS?

A common question is whether to perform a stress test before surgery. Based on the ACC/AHA guidelines,5 preoperative stress testing is not indicated solely to assess surgical risk if there is no other indication for it.

Stress testing can be used to determine whether the patient needs coronary revascularization. However, routine coronary revascularization is not recommended before noncardiac surgery exclusively to reduce perioperative cardiac events.

This conclusion is based on a landmark trial in which revascularization had no significant effect on outcomes.14 That trial included high-risk patients undergoing major vascular surgery who had greater than 70% stenosis of 1 or more major coronary arteries on angiography, randomized to either revascularization or no revascularization. It excluded patients with severe left main artery disease, ejection fraction less than 20%, and severe aortic stenosis. Results showed no differences in the rates of postoperative death, myocardial infarction, and stroke between the 2 groups. Furthermore, there was no postoperative survival difference during 5 years of follow-up.

Stress testing may be considered for patients with elevated risk and whose functional capacity is poor (< 4 metabolic equivalents) or unknown if it will change the management strategy. Another consideration affecting whether to perform stress testing is whether the surgery can be deferred for a month if the stress test is positive and a bare-metal coronary stent is placed, to allow for completion of dual antiplatelet therapy.

SHOULD WE ROUTINELY MONITOR TROPONIN AFTER SURGERY IN ASYMPTOMATIC PATIENTS?

Currently, the role of routine monitoring of troponin postoperatively in asymptomatic patients is unclear. The Canadian Cardiovascular Society12 recommends monitoring troponin in selected group of patients, eg, those with an RCRI score of 1 or higher, age 65 or older, a significant cardiac history, or elevated BNP preoperatively. However, at this point we do not have strong evidence regarding the implications of mild asymptomatic troponin elevation postoperatively and how to manage it. Two currently ongoing randomized controlled trials will answer those questions:

  • The Management of Myocardial Injury After Noncardiac Surgery (MANAGE) trial, comparing the use of dabigatran and omeprazole vs placebo in myocardial injury postoperatively
  • The Study of Ticagrelor Versus Aspirin Treatment in Patients With Myocardial Injury Post Major Non-cardiac Surgery (INTREPID).

Neither of the two cardiac risk assessment indexes most commonly used (Table 1)1,2 is completely accurate, nor is one superior to the other. To provide the most accurate assessment of cardiac risk, practitioners need to select the index most applicable to the circumstances of the individual patient.

CARDIAC COMPLICATIONS ARE INCREASING

The Revised Cardiac Risk Index and the National Surgical Quality Improvement Program index
About 5% of patients undergoing noncardiac surgery have a major cardiac complication within the first 30 postoperative days.3,4 This rate has been rising, primarily due to an increasing prevalence of cardiac comorbidities. Thus, accurate preoperative cardiac risk stratification is needed to assess the risk of perioperative major cardiac complications in all patients scheduled for noncardiac surgery. This information helps the perioperative team and patient to better weigh the benefits and risks of surgery and to optimize its timing and location (eg, inpatient vs outpatient surgery center).

CARDIAC RISK ASSESSMENT INDEXES

The 2 risk assessment indexes most often used are:

  • The Revised Cardiac Risk Index (RCRI)1
  • The National Surgical Quality Improvement Program (NSQIP) risk index, also known as the Gupta index.2

Both are endorsed by the American College of Cardiology (ACC) and the American Heart Association (AHA).5 The RCRI, introduced in 1999, is more commonly used, but the NSQIP, introduced in 2011, is based on a larger sample size.

Both indexes consider various factors in estimating the risk, with some overlap. The main outcome assessed in both indexes is the risk of a major cardiac event, ie, myocardial infarction or cardiac arrest. The RCRI outcome also includes ventricular fibrillation, complete heart block, and pulmonary edema, which may be sequelae to cardiac arrest and myocardial infarction. This difference in defined outcomes between the indexes is not likely to account for a significant variation in the prediction of risk; however, this is difficult to prove.

Each index defines myocardial infarction differently. The current clinical definition6 includes detection of a rise or fall of cardiac biomarker values (preferably cardiac troponins) with at least 1 value above the 99th percentile upper reference limit and at least 1 of the following:

  • Symptoms of ischemia
  • New ST-T wave changes or new left bundle branch block
  • New pathologic Q waves
  • Imaging evidence of new loss of viable myocardium tissue or new regional wall- motion abnormality
  • Finding of an intracoronary thrombus.

As seen in Table 1, the definition of myocardial infarction in NSQIP was one of the following: ST-segment elevation, new left bundle branch block, Q waves, or a troponin level greater than 3 times normal. Patients may have mild troponin leak of unknown significance without chest pain after surgery. This suggests that NSQIP may have overdiagnosed myocardial infarction.

USE IN CLINICAL PRACTICE

In clinical practice, which risk index is more accurate? Should clinicians become familiar with one index and keep using it? The 2014 ACC/AHA guidelines5 do not recommend one over the other, nor do they define the clinical situations that could lead to significant underestimation of risk.

The following are cases in which the indexes provide contradictory risk assessments.

Case 1. A 60-year-old man scheduled for surgery has diabetes mellitus, for which he takes insulin, and stable heart failure (left ventricular ejection fraction 40%). His RCRI score is 2, indicating an elevated 7% risk of cardiac complications; however, his NSQIP index is 0.31%. In this case, the NSQIP index probably underestimates the risk, as insulin-dependent diabetes and heart failure are not variables in the NSQIP index.

Case 2. A 60-year-old man who is partially functionally dependent and is on oxygen for severe chronic obstructive pulmonary disease is scheduled for craniotomy. His RCRI score is 0 (low risk), but his NSQIP index score (4.87%) indicates an elevated risk of cardiac complications based on his functional status, symptomatic chronic obstructive pulmonary disease, and high-risk surgery. In this case, the RCRI probably underestimates the risk.

These cases show that practitioners should not rely on just one index, but should rather decide which index to apply case by case. This avoids underestimating the risk. In patients with poor functional status and higher American Society of Anesthesiology class, the NSQIP index may provide a more accurate risk estimation than the RCRI. Patients with cardiomyopathy as well as those with insulin-dependent diabetes may be well assessed by the RCRI.

The following situations require additional caution when using these indexes, to avoid over- and underestimating cardiac risk.

 

 

PATIENTS WITH SEVERE AORTIC STENOSIS

Neither index lists severe aortic stenosis as a risk factor. The RCRI derivation and validation studies had only 5 patients with severe aortic stenosis, and the NSQIP validation study did not include any patients with aortic stenosis. Nevertheless, severe aortic stenosis increases the risk of cardiac complications in the perioperative period,7 making it important to consider in these patients.

Although patients with severe symptomatic aortic stenosis need valvular intervention before the surgery, patients who have asymptomatic severe aortic stenosis without associated cardiac dysfunction do not. Close hemodynamic monitoring during surgery is reasonable in the latter group.5,7

PATIENTS WITH RECENT STROKE

What would be the cardiac risk for a patient scheduled for elective hip surgery who has had a stroke within the last 3 months? If one applies both indexes, the cardiac risk comes to less than 1% (low risk) in both cases. However, this could be deceiving. A large study8 published in 2014 showed an elevated risk of cardiac complications in patients undergoing noncardiac surgery who had had an ischemic stroke within the previous 6 months; in the first 3 months, the odds ratio of developing a major adverse cardiovascular event was 14.23.This clearly overrides the traditional expert opinion-based evidence, which is that a time lapse of only 1 month after an ischemic stroke is safe for surgery.

PATIENTS WITH DIASTOLIC DYSFUNCTION

A 2016 meta-analysis and systematic review found that preoperative diastolic dysfunction was associated with higher rates of postoperative mortality and major adverse cardiac events, regardless of the left ventricular ejection fraction.9 However, the studies investigated included mostly patients undergoing cardiovascular surgeries. This raises the question of whether asymptomatic patients need echocardiography before surgery.

In a patient who has diastolic dysfunction, one should maintain adequate blood pressure control and euvolemia before the surgery and avoid hypertensive spikes in the immediate perioperative period, as hypertension is the worst enemy of those with diastolic dysfunction. Patients with atrial fibrillation may need more stringent heart rate control.

In a prospective study involving 1,005 consecutive vascular surgery patients, the 30-day cardiovascular event rate was highest in patients with symptomatic heart failure (49%), followed by those with asymptomatic systolic left ventricular dysfunction (23%), asymptomatic diastolic left ventricular dysfunction (18%), and normal left ventricular function (10%).10

Further studies are needed to determine whether the data obtained from the assessment of ventricular function in patients without signs or symptoms are significant enough to require updates to the criteria.

WHAT ABOUT THE ROLE OF BNP?

In a meta-analysis of 15 noncardiac surgery studies in 850 patients, preoperative B-type natriuretic peptide (BNP) levels independently predicted major adverse cardiac events, with levels greater than 372 pg/mL having a 36.7% incidence of major adverse cardiac events.11

A recent publication by the Canadian Cardiovascular Society12 strongly recommended measuring N-terminal-proBNP or BNP before noncardiac surgery to enhance perioperative cardiac risk estimation in patients who are age 65 or older, patients who are age 45 to 64 with significant cardiovascular disease, or patients who have an RCRI score of 1 or higher.

Further prospective randomized studies are needed to assess the utility of measuring BNP for preoperative cardiac risk evaluation.

PATIENTS WITH OBSTRUCTIVE SLEEP APNEA

Patients with obstructive sleep apnea scheduled for surgery under anesthesia have a higher risk of perioperative complications than patients without the disease, including higher rates of cardiac complications and atrial fibrillation. However, the evidence is insufficient to support canceling or delaying surgery in patients with suspected obstructive sleep apnea.

After comorbid conditions are optimally treated, patients with obstructive sleep apnea can proceed to surgery, provided strategies for mitigating complications are implemented.13

 

 

TO STRESS OR NOT TO STRESS?

A common question is whether to perform a stress test before surgery. Based on the ACC/AHA guidelines,5 preoperative stress testing is not indicated solely to assess surgical risk if there is no other indication for it.

Stress testing can be used to determine whether the patient needs coronary revascularization. However, routine coronary revascularization is not recommended before noncardiac surgery exclusively to reduce perioperative cardiac events.

This conclusion is based on a landmark trial in which revascularization had no significant effect on outcomes.14 That trial included high-risk patients undergoing major vascular surgery who had greater than 70% stenosis of 1 or more major coronary arteries on angiography, randomized to either revascularization or no revascularization. It excluded patients with severe left main artery disease, ejection fraction less than 20%, and severe aortic stenosis. Results showed no differences in the rates of postoperative death, myocardial infarction, and stroke between the 2 groups. Furthermore, there was no postoperative survival difference during 5 years of follow-up.

Stress testing may be considered for patients with elevated risk and whose functional capacity is poor (< 4 metabolic equivalents) or unknown if it will change the management strategy. Another consideration affecting whether to perform stress testing is whether the surgery can be deferred for a month if the stress test is positive and a bare-metal coronary stent is placed, to allow for completion of dual antiplatelet therapy.

SHOULD WE ROUTINELY MONITOR TROPONIN AFTER SURGERY IN ASYMPTOMATIC PATIENTS?

Currently, the role of routine monitoring of troponin postoperatively in asymptomatic patients is unclear. The Canadian Cardiovascular Society12 recommends monitoring troponin in selected group of patients, eg, those with an RCRI score of 1 or higher, age 65 or older, a significant cardiac history, or elevated BNP preoperatively. However, at this point we do not have strong evidence regarding the implications of mild asymptomatic troponin elevation postoperatively and how to manage it. Two currently ongoing randomized controlled trials will answer those questions:

  • The Management of Myocardial Injury After Noncardiac Surgery (MANAGE) trial, comparing the use of dabigatran and omeprazole vs placebo in myocardial injury postoperatively
  • The Study of Ticagrelor Versus Aspirin Treatment in Patients With Myocardial Injury Post Major Non-cardiac Surgery (INTREPID).
References
  1. Lee TH, Marcantonio ER, Mangione CM, et al. Derivation and prospective validation of a simple index for prediction of cardiac risk of major noncardiac surgery. Circulation 1999; 100:1043–1049.
  2. Gupta PK, Gupta H, Sundaram A, et al. Development and validation of a risk calculator for prediction of cardiac risk after surgery. Circulation 2011; 124:381–387.
  3. Devereaux PJ, Sessler DI. Cardiac complications in patients undergoing major noncardiac surgery. N Engl J Med 2015; 373:2258–2269.
  4. Smilowitz NR, Gupta N, Ramakrishna H, Guo Y, Berger JS, Bangalore S. Perioperative major adverse cardiovascular and cerebrovascular events associated with noncardiac surgery. JAMA Cardiol 2017; 2:181–187.
  5. Fleisher LA, Fleischmann KE, Auerbach AD, et al; American College of Cardiology; American Heart Association. 2014 ACC/AHA guideline on perioperative cardiovascular evaluation and management of patients undergoing noncardiac surgery: a report of the American College of Cardiology/American Heart Association Task Force on practice guidelines. J Am Coll Cardiol 2014; 64:e77–e137 [Simultaneous publication: Circulation 2014; 130:2215–2245].
  6. Thygesen K, Alpert JS, Jaffe AS, et al, for the Joint ESC/ACCF/AHA/WHF Task Force for the Universal Definition of Myocardial Infarction. Third universal definition of myocardial infarction. Circulation 2012; 126:2020–2035.
  7. Tashiro T, Pislaru SV, Blustin JM, et al. Perioperative risk of major non-cardiac surgery in patients with severe aortic stenosis: a reappraisal in contemporary practice. Eur Heart J 2014; 35:2372–2381.
  8. Jørgensen ME, Torp-Pedersen C, Gislason GH, et al. Time elapsed after ischemic stroke and risk of adverse cardiovascular events and mortality following elective noncardiac surgery. JAMA 2014; 312:269–277.
  9. Kaw R, Hernandez AV, Pasupuleti V, et al; Cardiovascular Meta-analyses Research Group. Effect of diastolic dysfunction on postoperative outcomes after cardiovascular surgery: a systematic review and meta-analysis. J Thorac Cardiovasc Surg 2016; 152:1142–1153.
  10. Flu WJ, van Kuijk JP, Hoeks SE, et al. Prognostic implications of asymptomatic left ventricular dysfunction in patients undergoing vascular surgery. Anesthesiology 2010; 112:1316–1324.
  11. Rodseth R, Lurati Buse G, Bolliger D, et al. The predictive ability of pre-operative B-type natriuretic peptide in vascular patients for major adverse cardiac events: an individual patient data meta-analysis. J Am Coll Cardiol 2011; 58:522–529.
  12. Duceppe E, Parlow J, MacDonald P, et al. Canadian Cardiovascular Society Guidelines on perioperative cardiac risk assessment and management for patients who undergo noncardiac surgery. Can J Cardiol 2017; 33:17–32.
  13. Chung F, Memtsoudis SG, Ramachandran SK, et al. Society of Anesthesia and Sleep Medicine guidelines on preoperative screening and assessment of adult patients with obstructive sleep apnea. Anesth Analg 2016; 123:452–473.
  14. McFalls EO, Ward HB, Moritz TE, et al. Coronary-artery revascularization before elective major vascular surgery. N Engl J Med 2004; 351:2795–2804.
References
  1. Lee TH, Marcantonio ER, Mangione CM, et al. Derivation and prospective validation of a simple index for prediction of cardiac risk of major noncardiac surgery. Circulation 1999; 100:1043–1049.
  2. Gupta PK, Gupta H, Sundaram A, et al. Development and validation of a risk calculator for prediction of cardiac risk after surgery. Circulation 2011; 124:381–387.
  3. Devereaux PJ, Sessler DI. Cardiac complications in patients undergoing major noncardiac surgery. N Engl J Med 2015; 373:2258–2269.
  4. Smilowitz NR, Gupta N, Ramakrishna H, Guo Y, Berger JS, Bangalore S. Perioperative major adverse cardiovascular and cerebrovascular events associated with noncardiac surgery. JAMA Cardiol 2017; 2:181–187.
  5. Fleisher LA, Fleischmann KE, Auerbach AD, et al; American College of Cardiology; American Heart Association. 2014 ACC/AHA guideline on perioperative cardiovascular evaluation and management of patients undergoing noncardiac surgery: a report of the American College of Cardiology/American Heart Association Task Force on practice guidelines. J Am Coll Cardiol 2014; 64:e77–e137 [Simultaneous publication: Circulation 2014; 130:2215–2245].
  6. Thygesen K, Alpert JS, Jaffe AS, et al, for the Joint ESC/ACCF/AHA/WHF Task Force for the Universal Definition of Myocardial Infarction. Third universal definition of myocardial infarction. Circulation 2012; 126:2020–2035.
  7. Tashiro T, Pislaru SV, Blustin JM, et al. Perioperative risk of major non-cardiac surgery in patients with severe aortic stenosis: a reappraisal in contemporary practice. Eur Heart J 2014; 35:2372–2381.
  8. Jørgensen ME, Torp-Pedersen C, Gislason GH, et al. Time elapsed after ischemic stroke and risk of adverse cardiovascular events and mortality following elective noncardiac surgery. JAMA 2014; 312:269–277.
  9. Kaw R, Hernandez AV, Pasupuleti V, et al; Cardiovascular Meta-analyses Research Group. Effect of diastolic dysfunction on postoperative outcomes after cardiovascular surgery: a systematic review and meta-analysis. J Thorac Cardiovasc Surg 2016; 152:1142–1153.
  10. Flu WJ, van Kuijk JP, Hoeks SE, et al. Prognostic implications of asymptomatic left ventricular dysfunction in patients undergoing vascular surgery. Anesthesiology 2010; 112:1316–1324.
  11. Rodseth R, Lurati Buse G, Bolliger D, et al. The predictive ability of pre-operative B-type natriuretic peptide in vascular patients for major adverse cardiac events: an individual patient data meta-analysis. J Am Coll Cardiol 2011; 58:522–529.
  12. Duceppe E, Parlow J, MacDonald P, et al. Canadian Cardiovascular Society Guidelines on perioperative cardiac risk assessment and management for patients who undergo noncardiac surgery. Can J Cardiol 2017; 33:17–32.
  13. Chung F, Memtsoudis SG, Ramachandran SK, et al. Society of Anesthesia and Sleep Medicine guidelines on preoperative screening and assessment of adult patients with obstructive sleep apnea. Anesth Analg 2016; 123:452–473.
  14. McFalls EO, Ward HB, Moritz TE, et al. Coronary-artery revascularization before elective major vascular surgery. N Engl J Med 2004; 351:2795–2804.
Issue
Cleveland Clinic Journal of Medicine - 85(1)
Issue
Cleveland Clinic Journal of Medicine - 85(1)
Page Number
35-39
Page Number
35-39
Publications
Publications
Topics
Article Type
Display Headline
Do cardiac risk stratification indexes accurately estimate perioperative risk in noncardiac surgery patients?
Display Headline
Do cardiac risk stratification indexes accurately estimate perioperative risk in noncardiac surgery patients?
Legacy Keywords
risk stratification, perioperative, cardiac, risk indexes, perioperative, surgery, Revised Cardiac Risk Index, RCRI, National Surgical Quality Improvement Program, NSQIP, myocardial infarction, MI, Rohan Mandaliya, Geno Merli
Legacy Keywords
risk stratification, perioperative, cardiac, risk indexes, perioperative, surgery, Revised Cardiac Risk Index, RCRI, National Surgical Quality Improvement Program, NSQIP, myocardial infarction, MI, Rohan Mandaliya, Geno Merli
Sections
Disallow All Ads
Content Gating
No Gating (article Unlocked/Free)
Alternative CME
Disqus Comments
Default
Article PDF Media

What is the hepatitis B vaccination regimen in chronic kidney disease?

Article Type
Changed
Thu, 07/19/2018 - 10:45
Display Headline
What is the hepatitis B vaccination regimen in chronic kidney disease?

For patients age 16 and older with advanced chronic kidney disease (CKD), including those undergoing hemodialysis, we recommend a higher dose of hepatitis B virus (HBV) vaccine, more doses, or both. Vaccination with a higher dose may improve the immune response. The hepatitis B surface antibody (anti-HBs) titer should be monitored 1 to 2 months after completion of the vaccination schedule and annually thereafter, with a target titer of 10 IU/mL or greater. For patients who do not develop a protective antibody titer after completing the initial vaccination schedule, the vaccination schedule should be repeated.

RECOMMENDED DOSES AND SCHEDULES

Recommendation 1

Give higher vaccine doses, increase the number of doses, or both.

Recommended vaccination regimens for hepatitis B virus in chronic kidney disease
Rationale. Patients with CKD, especially those on hemodialysis, are in an immunocompromised state and thus are less likely to achieve protective anti-HBs levels after vaccination with standard dosages.1–3 Two main vaccine formulations are available (Table 1). Recombivax-HB contains 40 µg/mL and is given in a 3-dose schedule at 0, 1, and 6 months. Engerix-B contains a standard dose of 20 µg/mL and should be given in a 4-dose schedule at double the standard dose (ie, a total of 40 µg/mL). Both regimens are recommended in the 2017 update of the United States Advisory Committee on Immunization Practices (ACIP) recommendations for adult immunization schedule.4

Recommendation 2

A 4-dose regimen may provide a better antibody response than a 3-dose regimen. (Note: This recommendation applies only to Engerix-B; 4 doses of Recombivax-HB would be an off-label use.)

Rationale. The US Centers for Disease Control and Prevention reported that after completion of a 3-dose vaccination schedule, the median proportion of patients developing a protective antibody response was 64% (range 34%–88%) vs a median of 86% (range 40%–98%) after a 4-dose schedule.3

Lacson et al5 compared antibody response rates after 3 doses of Recombivax-HB and after 4 doses of Engerix-B and found a better response rate with the 4-dose schedule. The rate of persistent protective anti-HBs titer after 1 year was 77% for Engerix-B vs 53% for Recombivax-HB.

Agarwal et al6 evaluated response rates in patients who had mild CKD (serum creatinine levels 1.5–3.0 mg/dL), moderate CKD (creatinine 3.0–6.0 mg/dL), and severe CKD (creatinine > 6.0 mg/dL). The seroconversion rates after 3 doses of 40-μg HBV vaccine were 87.5% in those with mild CKD, 66.6% in those with moderate CKD, and 35.7% in those with severe disease. After a fourth dose, rates improved significantly to 100%, 77%, and 36.4%, respectively.

Recommendation 3

In patients with CKD, vaccination should be done early, before they become dependent on hemodialysis.

Rationale. Patients with advanced CKD may have a lower seroconversion rate. Fraser et al7 found that after a 4-dose series, the seroprotection rate in adult prehemodialysis patients with serum creatinine levels of 4 mg/dL or less was 86%, compared with 37% in patients with serum creatinine levels above 4 mg/dL, of whom 88% were on hemodialysis.7

In a 2003 prospective cohort study by DaRoza et al,8 patients with higher levels of kidney function were more likely to respond to HBV vaccination, and the level of kidney function was found to be an independent predictor of seroconversion.8

A 2012 prospective study by Ghadiani et al9 compared seroconversion rates in patients with stage 3 or 4 CKD vs patients on hemodialysis, with medical staff as controls. The authors reported seroprotection rates of 26.1% in patients on hemodialysis, 55.2% in patients with stage 3 or 4 CKD, and 96.2% in controls. They concluded that vaccination is more likely to induce seroconversion in earlier stages of kidney disease.9

 

 

MONITORING THE RESPONSE TO VACCINATION AND REVACCINATION

Testing after vaccination is recommended to determine response. Testing should be done 1 to 2 months after the last dose of the vaccination schedule.1–3 Anti-HBs levels 10 IU/mL and greater are considered protective.10

Revaccination with a full vaccination series is recommended for patients who do not develop adequate levels of protective antibodies after completion of the vaccination schedule.2 Reported response rates to revaccination have varied from 40% to 50% after 2 or 3 additional intramuscular  doses of 40 µg, to 64% after 4 additional intramuscular doses of 10 µg.3 Serologic testing should be repeated after the last dose of the vaccination series, as serologic testing after only 1 or 2 additional doses appears to be no more cost-effective.2,3

To the best of our knowledge, no data exist to indicate that in nonresponders, further doses given after completion of 2 full vaccination schedules would induce an antibody response.

ANTIBODY PERSISTENCE AND BOOSTER DOSES

Antibody levels fall with time in patients on hemodialysis. Limited data suggest that in patients who respond to the primary vaccination series, antibodies remain detectable for 6 months in 80% to 100% (median 100%) of patients and for 12 months in 58% to 100% (median 70%) of patients.3 The need for booster doses should be assessed by annual monitoring.2,11 Booster doses should be given when the anti-HBs titer declines to below 10 IU/mL. Limited data indicate that nearly all such patients would respond to a booster dose.3

OTHER WAYS TO IMPROVE VACCINE RESPONSE

Other strategies to improve vaccine response, such as the addition of adjuvants or immunostimulants, have shown variable success.12 Intradermal HBV vaccination in patients on chronic hemodialysis has also been proposed. The efficacy of intradermal vaccination may be related to the dense network of immunologic dendritic cells within the dermis. After intradermal administration, the antigen is taken up by dendritic cells residing in the dermis, which mature and travel to the regional lymph node where further immunostimulation takes place.13

In a systematic review of four prospective trials with a total of 204 hemodialysis patients,13 a significantly higher proportion of patients achieved seroconversion with intradermal HBV vaccine administration than with intramuscular administration. The authors concluded that the intradermal route in primary nonresponders undergoing hemodialysis provides an effective alternative to the intramuscular route to protect against HBV infection in this highly susceptible population.

Additional well-designed, double-blinded, randomized trials are needed to establish clear guidelines on intradermal HBV vaccine dosing and vaccination schedules.

References
  1. Grzegorzewska AE. Hepatitis B vaccination in chronic kidney disease: review of evidence in non-dialyzed patients. Hepat Mon 2012; 12:e7359.
  2. Chi C, Patel P, Pilishvili T, Moore M, Murphy T, Strikas R. Guidelines for vaccinating kidney dialysis patients and patients with chronic kidney disease. www.cdc.gov/dialysis/PDFs/Vaccinating_Dialysis_Patients_and_Patients_dec2012.pdf. Accessed September 6, 2017.
  3. Recommendations for preventing transmission of infections among chronic hemodialysis patients. MMWR Recomm Rep 2001; 50:1–43.
  4. Kim DK, Riley LE, Harriman KH, Hunter P, Bridges CB; Advisory Committee on Immunization Practices. Recommended immunization schedule for adults aged 19 years or older, United States, 2017. Ann Intern Med 2017; 166:209–219.
  5. Lacson E, Teng M, Ong J, Vienneau L, Ofsthun N, Lazarus JM. Antibody response to Engerix-B and Recombivax-HB hepatitis B vaccination in end-stage renal disease. Hemodialysis international. Hemodial Int 2005; 9:367–375.
  6. Agarwal SK, Irshad M, Dash SC. Comparison of two schedules of hepatitis B vaccination in patients with mild, moderate and severe renal failure. J Assoc Physicians India 1999; 47:183–185.
  7. Fraser GM, Ochana N, Fenyves D, et al. Increasing serum creatinine and age reduce the response to hepatitis B vaccine in renal failure patients. J Hepatol 1994; 21:450–454.
  8. DaRoza G, Loewen A, Djurdjev O, et al. Stage of chronic kidney disease predicts seroconversion after hepatitis B immunization: earlier is better. Am J Kidney Dis 2003; 42:1184–1192.
  9. Ghadiani MH, Besharati S, Mousavinasab N, Jalalzadeh M. Response rates to HB vaccine in CKD stages 3-4 and hemodialysis patients. J Res Med Sci 2012; 17:527–533.
  10. Jack AD, Hall AJ, Maine N, Mendy M, Whittle HC. What level of hepatitis B antibody is protective? J Infect Dis 1999; 179:489–492.
  11. Guidelines for vaccination in patients with chronic kidney disease. Indian J Nephrol 2016; 26(suppl 1):S15–S18.
  12. Somi MH, Hajipour B. Improving hepatitis B vaccine efficacy in end-stage renal diseases patients and role of adjuvants. ISRN Gastroenterol 2012; 2012:960413.
  13. Yousaf F, Gandham S, Galler M, Spinowitz B, Charytan C. Systematic review of the efficacy and safety of intradermal versus intramuscular hepatitis B vaccination in end-stage renal disease population unresponsive to primary vaccination series. Ren Fail 2015; 37:1080–1088.
Article PDF
Author and Disclosure Information

Kheng Yong Ong, BSc (Pharm) (Hons)
Pharmacist, Department of Pharmacy, Singapore General Hospital, Singapore

Hong Yun Wong, BSc (Pharm) (Hons)
Pharmacist, Department of Pharmacy, Khoo Teck Puat Hospital, Singapore

Giat Yeng Khee, PharmD
Senior Clinical Pharmacist, Department of Pharmacy, Singapore General Hospital, Singapore

Address: Kheng Yong Ong, BSc, Department of Pharmacy, Singapore General Hospital, Outram Road, Singapore 169608; ong.kheng.yong@sgh.com.sg

Issue
Cleveland Clinic Journal of Medicine - 85(1)
Publications
Topics
Page Number
32-34
Legacy Keywords
hepatitis B, HBV, vaccination, chronic kidney disease, CKD, Engerix-B, Recombivax-HB, Kheng Yong Ong, Hong Yun Wong, Giat Yeng Khee
Sections
Author and Disclosure Information

Kheng Yong Ong, BSc (Pharm) (Hons)
Pharmacist, Department of Pharmacy, Singapore General Hospital, Singapore

Hong Yun Wong, BSc (Pharm) (Hons)
Pharmacist, Department of Pharmacy, Khoo Teck Puat Hospital, Singapore

Giat Yeng Khee, PharmD
Senior Clinical Pharmacist, Department of Pharmacy, Singapore General Hospital, Singapore

Address: Kheng Yong Ong, BSc, Department of Pharmacy, Singapore General Hospital, Outram Road, Singapore 169608; ong.kheng.yong@sgh.com.sg

Author and Disclosure Information

Kheng Yong Ong, BSc (Pharm) (Hons)
Pharmacist, Department of Pharmacy, Singapore General Hospital, Singapore

Hong Yun Wong, BSc (Pharm) (Hons)
Pharmacist, Department of Pharmacy, Khoo Teck Puat Hospital, Singapore

Giat Yeng Khee, PharmD
Senior Clinical Pharmacist, Department of Pharmacy, Singapore General Hospital, Singapore

Address: Kheng Yong Ong, BSc, Department of Pharmacy, Singapore General Hospital, Outram Road, Singapore 169608; ong.kheng.yong@sgh.com.sg

Article PDF
Article PDF
Related Articles

For patients age 16 and older with advanced chronic kidney disease (CKD), including those undergoing hemodialysis, we recommend a higher dose of hepatitis B virus (HBV) vaccine, more doses, or both. Vaccination with a higher dose may improve the immune response. The hepatitis B surface antibody (anti-HBs) titer should be monitored 1 to 2 months after completion of the vaccination schedule and annually thereafter, with a target titer of 10 IU/mL or greater. For patients who do not develop a protective antibody titer after completing the initial vaccination schedule, the vaccination schedule should be repeated.

RECOMMENDED DOSES AND SCHEDULES

Recommendation 1

Give higher vaccine doses, increase the number of doses, or both.

Recommended vaccination regimens for hepatitis B virus in chronic kidney disease
Rationale. Patients with CKD, especially those on hemodialysis, are in an immunocompromised state and thus are less likely to achieve protective anti-HBs levels after vaccination with standard dosages.1–3 Two main vaccine formulations are available (Table 1). Recombivax-HB contains 40 µg/mL and is given in a 3-dose schedule at 0, 1, and 6 months. Engerix-B contains a standard dose of 20 µg/mL and should be given in a 4-dose schedule at double the standard dose (ie, a total of 40 µg/mL). Both regimens are recommended in the 2017 update of the United States Advisory Committee on Immunization Practices (ACIP) recommendations for adult immunization schedule.4

Recommendation 2

A 4-dose regimen may provide a better antibody response than a 3-dose regimen. (Note: This recommendation applies only to Engerix-B; 4 doses of Recombivax-HB would be an off-label use.)

Rationale. The US Centers for Disease Control and Prevention reported that after completion of a 3-dose vaccination schedule, the median proportion of patients developing a protective antibody response was 64% (range 34%–88%) vs a median of 86% (range 40%–98%) after a 4-dose schedule.3

Lacson et al5 compared antibody response rates after 3 doses of Recombivax-HB and after 4 doses of Engerix-B and found a better response rate with the 4-dose schedule. The rate of persistent protective anti-HBs titer after 1 year was 77% for Engerix-B vs 53% for Recombivax-HB.

Agarwal et al6 evaluated response rates in patients who had mild CKD (serum creatinine levels 1.5–3.0 mg/dL), moderate CKD (creatinine 3.0–6.0 mg/dL), and severe CKD (creatinine > 6.0 mg/dL). The seroconversion rates after 3 doses of 40-μg HBV vaccine were 87.5% in those with mild CKD, 66.6% in those with moderate CKD, and 35.7% in those with severe disease. After a fourth dose, rates improved significantly to 100%, 77%, and 36.4%, respectively.

Recommendation 3

In patients with CKD, vaccination should be done early, before they become dependent on hemodialysis.

Rationale. Patients with advanced CKD may have a lower seroconversion rate. Fraser et al7 found that after a 4-dose series, the seroprotection rate in adult prehemodialysis patients with serum creatinine levels of 4 mg/dL or less was 86%, compared with 37% in patients with serum creatinine levels above 4 mg/dL, of whom 88% were on hemodialysis.7

In a 2003 prospective cohort study by DaRoza et al,8 patients with higher levels of kidney function were more likely to respond to HBV vaccination, and the level of kidney function was found to be an independent predictor of seroconversion.8

A 2012 prospective study by Ghadiani et al9 compared seroconversion rates in patients with stage 3 or 4 CKD vs patients on hemodialysis, with medical staff as controls. The authors reported seroprotection rates of 26.1% in patients on hemodialysis, 55.2% in patients with stage 3 or 4 CKD, and 96.2% in controls. They concluded that vaccination is more likely to induce seroconversion in earlier stages of kidney disease.9

 

 

MONITORING THE RESPONSE TO VACCINATION AND REVACCINATION

Testing after vaccination is recommended to determine response. Testing should be done 1 to 2 months after the last dose of the vaccination schedule.1–3 Anti-HBs levels 10 IU/mL and greater are considered protective.10

Revaccination with a full vaccination series is recommended for patients who do not develop adequate levels of protective antibodies after completion of the vaccination schedule.2 Reported response rates to revaccination have varied from 40% to 50% after 2 or 3 additional intramuscular  doses of 40 µg, to 64% after 4 additional intramuscular doses of 10 µg.3 Serologic testing should be repeated after the last dose of the vaccination series, as serologic testing after only 1 or 2 additional doses appears to be no more cost-effective.2,3

To the best of our knowledge, no data exist to indicate that in nonresponders, further doses given after completion of 2 full vaccination schedules would induce an antibody response.

ANTIBODY PERSISTENCE AND BOOSTER DOSES

Antibody levels fall with time in patients on hemodialysis. Limited data suggest that in patients who respond to the primary vaccination series, antibodies remain detectable for 6 months in 80% to 100% (median 100%) of patients and for 12 months in 58% to 100% (median 70%) of patients.3 The need for booster doses should be assessed by annual monitoring.2,11 Booster doses should be given when the anti-HBs titer declines to below 10 IU/mL. Limited data indicate that nearly all such patients would respond to a booster dose.3

OTHER WAYS TO IMPROVE VACCINE RESPONSE

Other strategies to improve vaccine response, such as the addition of adjuvants or immunostimulants, have shown variable success.12 Intradermal HBV vaccination in patients on chronic hemodialysis has also been proposed. The efficacy of intradermal vaccination may be related to the dense network of immunologic dendritic cells within the dermis. After intradermal administration, the antigen is taken up by dendritic cells residing in the dermis, which mature and travel to the regional lymph node where further immunostimulation takes place.13

In a systematic review of four prospective trials with a total of 204 hemodialysis patients,13 a significantly higher proportion of patients achieved seroconversion with intradermal HBV vaccine administration than with intramuscular administration. The authors concluded that the intradermal route in primary nonresponders undergoing hemodialysis provides an effective alternative to the intramuscular route to protect against HBV infection in this highly susceptible population.

Additional well-designed, double-blinded, randomized trials are needed to establish clear guidelines on intradermal HBV vaccine dosing and vaccination schedules.

For patients age 16 and older with advanced chronic kidney disease (CKD), including those undergoing hemodialysis, we recommend a higher dose of hepatitis B virus (HBV) vaccine, more doses, or both. Vaccination with a higher dose may improve the immune response. The hepatitis B surface antibody (anti-HBs) titer should be monitored 1 to 2 months after completion of the vaccination schedule and annually thereafter, with a target titer of 10 IU/mL or greater. For patients who do not develop a protective antibody titer after completing the initial vaccination schedule, the vaccination schedule should be repeated.

RECOMMENDED DOSES AND SCHEDULES

Recommendation 1

Give higher vaccine doses, increase the number of doses, or both.

Recommended vaccination regimens for hepatitis B virus in chronic kidney disease
Rationale. Patients with CKD, especially those on hemodialysis, are in an immunocompromised state and thus are less likely to achieve protective anti-HBs levels after vaccination with standard dosages.1–3 Two main vaccine formulations are available (Table 1). Recombivax-HB contains 40 µg/mL and is given in a 3-dose schedule at 0, 1, and 6 months. Engerix-B contains a standard dose of 20 µg/mL and should be given in a 4-dose schedule at double the standard dose (ie, a total of 40 µg/mL). Both regimens are recommended in the 2017 update of the United States Advisory Committee on Immunization Practices (ACIP) recommendations for adult immunization schedule.4

Recommendation 2

A 4-dose regimen may provide a better antibody response than a 3-dose regimen. (Note: This recommendation applies only to Engerix-B; 4 doses of Recombivax-HB would be an off-label use.)

Rationale. The US Centers for Disease Control and Prevention reported that after completion of a 3-dose vaccination schedule, the median proportion of patients developing a protective antibody response was 64% (range 34%–88%) vs a median of 86% (range 40%–98%) after a 4-dose schedule.3

Lacson et al5 compared antibody response rates after 3 doses of Recombivax-HB and after 4 doses of Engerix-B and found a better response rate with the 4-dose schedule. The rate of persistent protective anti-HBs titer after 1 year was 77% for Engerix-B vs 53% for Recombivax-HB.

Agarwal et al6 evaluated response rates in patients who had mild CKD (serum creatinine levels 1.5–3.0 mg/dL), moderate CKD (creatinine 3.0–6.0 mg/dL), and severe CKD (creatinine > 6.0 mg/dL). The seroconversion rates after 3 doses of 40-μg HBV vaccine were 87.5% in those with mild CKD, 66.6% in those with moderate CKD, and 35.7% in those with severe disease. After a fourth dose, rates improved significantly to 100%, 77%, and 36.4%, respectively.

Recommendation 3

In patients with CKD, vaccination should be done early, before they become dependent on hemodialysis.

Rationale. Patients with advanced CKD may have a lower seroconversion rate. Fraser et al7 found that after a 4-dose series, the seroprotection rate in adult prehemodialysis patients with serum creatinine levels of 4 mg/dL or less was 86%, compared with 37% in patients with serum creatinine levels above 4 mg/dL, of whom 88% were on hemodialysis.7

In a 2003 prospective cohort study by DaRoza et al,8 patients with higher levels of kidney function were more likely to respond to HBV vaccination, and the level of kidney function was found to be an independent predictor of seroconversion.8

A 2012 prospective study by Ghadiani et al9 compared seroconversion rates in patients with stage 3 or 4 CKD vs patients on hemodialysis, with medical staff as controls. The authors reported seroprotection rates of 26.1% in patients on hemodialysis, 55.2% in patients with stage 3 or 4 CKD, and 96.2% in controls. They concluded that vaccination is more likely to induce seroconversion in earlier stages of kidney disease.9

 

 

MONITORING THE RESPONSE TO VACCINATION AND REVACCINATION

Testing after vaccination is recommended to determine response. Testing should be done 1 to 2 months after the last dose of the vaccination schedule.1–3 Anti-HBs levels 10 IU/mL and greater are considered protective.10

Revaccination with a full vaccination series is recommended for patients who do not develop adequate levels of protective antibodies after completion of the vaccination schedule.2 Reported response rates to revaccination have varied from 40% to 50% after 2 or 3 additional intramuscular  doses of 40 µg, to 64% after 4 additional intramuscular doses of 10 µg.3 Serologic testing should be repeated after the last dose of the vaccination series, as serologic testing after only 1 or 2 additional doses appears to be no more cost-effective.2,3

To the best of our knowledge, no data exist to indicate that in nonresponders, further doses given after completion of 2 full vaccination schedules would induce an antibody response.

ANTIBODY PERSISTENCE AND BOOSTER DOSES

Antibody levels fall with time in patients on hemodialysis. Limited data suggest that in patients who respond to the primary vaccination series, antibodies remain detectable for 6 months in 80% to 100% (median 100%) of patients and for 12 months in 58% to 100% (median 70%) of patients.3 The need for booster doses should be assessed by annual monitoring.2,11 Booster doses should be given when the anti-HBs titer declines to below 10 IU/mL. Limited data indicate that nearly all such patients would respond to a booster dose.3

OTHER WAYS TO IMPROVE VACCINE RESPONSE

Other strategies to improve vaccine response, such as the addition of adjuvants or immunostimulants, have shown variable success.12 Intradermal HBV vaccination in patients on chronic hemodialysis has also been proposed. The efficacy of intradermal vaccination may be related to the dense network of immunologic dendritic cells within the dermis. After intradermal administration, the antigen is taken up by dendritic cells residing in the dermis, which mature and travel to the regional lymph node where further immunostimulation takes place.13

In a systematic review of four prospective trials with a total of 204 hemodialysis patients,13 a significantly higher proportion of patients achieved seroconversion with intradermal HBV vaccine administration than with intramuscular administration. The authors concluded that the intradermal route in primary nonresponders undergoing hemodialysis provides an effective alternative to the intramuscular route to protect against HBV infection in this highly susceptible population.

Additional well-designed, double-blinded, randomized trials are needed to establish clear guidelines on intradermal HBV vaccine dosing and vaccination schedules.

References
  1. Grzegorzewska AE. Hepatitis B vaccination in chronic kidney disease: review of evidence in non-dialyzed patients. Hepat Mon 2012; 12:e7359.
  2. Chi C, Patel P, Pilishvili T, Moore M, Murphy T, Strikas R. Guidelines for vaccinating kidney dialysis patients and patients with chronic kidney disease. www.cdc.gov/dialysis/PDFs/Vaccinating_Dialysis_Patients_and_Patients_dec2012.pdf. Accessed September 6, 2017.
  3. Recommendations for preventing transmission of infections among chronic hemodialysis patients. MMWR Recomm Rep 2001; 50:1–43.
  4. Kim DK, Riley LE, Harriman KH, Hunter P, Bridges CB; Advisory Committee on Immunization Practices. Recommended immunization schedule for adults aged 19 years or older, United States, 2017. Ann Intern Med 2017; 166:209–219.
  5. Lacson E, Teng M, Ong J, Vienneau L, Ofsthun N, Lazarus JM. Antibody response to Engerix-B and Recombivax-HB hepatitis B vaccination in end-stage renal disease. Hemodialysis international. Hemodial Int 2005; 9:367–375.
  6. Agarwal SK, Irshad M, Dash SC. Comparison of two schedules of hepatitis B vaccination in patients with mild, moderate and severe renal failure. J Assoc Physicians India 1999; 47:183–185.
  7. Fraser GM, Ochana N, Fenyves D, et al. Increasing serum creatinine and age reduce the response to hepatitis B vaccine in renal failure patients. J Hepatol 1994; 21:450–454.
  8. DaRoza G, Loewen A, Djurdjev O, et al. Stage of chronic kidney disease predicts seroconversion after hepatitis B immunization: earlier is better. Am J Kidney Dis 2003; 42:1184–1192.
  9. Ghadiani MH, Besharati S, Mousavinasab N, Jalalzadeh M. Response rates to HB vaccine in CKD stages 3-4 and hemodialysis patients. J Res Med Sci 2012; 17:527–533.
  10. Jack AD, Hall AJ, Maine N, Mendy M, Whittle HC. What level of hepatitis B antibody is protective? J Infect Dis 1999; 179:489–492.
  11. Guidelines for vaccination in patients with chronic kidney disease. Indian J Nephrol 2016; 26(suppl 1):S15–S18.
  12. Somi MH, Hajipour B. Improving hepatitis B vaccine efficacy in end-stage renal diseases patients and role of adjuvants. ISRN Gastroenterol 2012; 2012:960413.
  13. Yousaf F, Gandham S, Galler M, Spinowitz B, Charytan C. Systematic review of the efficacy and safety of intradermal versus intramuscular hepatitis B vaccination in end-stage renal disease population unresponsive to primary vaccination series. Ren Fail 2015; 37:1080–1088.
References
  1. Grzegorzewska AE. Hepatitis B vaccination in chronic kidney disease: review of evidence in non-dialyzed patients. Hepat Mon 2012; 12:e7359.
  2. Chi C, Patel P, Pilishvili T, Moore M, Murphy T, Strikas R. Guidelines for vaccinating kidney dialysis patients and patients with chronic kidney disease. www.cdc.gov/dialysis/PDFs/Vaccinating_Dialysis_Patients_and_Patients_dec2012.pdf. Accessed September 6, 2017.
  3. Recommendations for preventing transmission of infections among chronic hemodialysis patients. MMWR Recomm Rep 2001; 50:1–43.
  4. Kim DK, Riley LE, Harriman KH, Hunter P, Bridges CB; Advisory Committee on Immunization Practices. Recommended immunization schedule for adults aged 19 years or older, United States, 2017. Ann Intern Med 2017; 166:209–219.
  5. Lacson E, Teng M, Ong J, Vienneau L, Ofsthun N, Lazarus JM. Antibody response to Engerix-B and Recombivax-HB hepatitis B vaccination in end-stage renal disease. Hemodialysis international. Hemodial Int 2005; 9:367–375.
  6. Agarwal SK, Irshad M, Dash SC. Comparison of two schedules of hepatitis B vaccination in patients with mild, moderate and severe renal failure. J Assoc Physicians India 1999; 47:183–185.
  7. Fraser GM, Ochana N, Fenyves D, et al. Increasing serum creatinine and age reduce the response to hepatitis B vaccine in renal failure patients. J Hepatol 1994; 21:450–454.
  8. DaRoza G, Loewen A, Djurdjev O, et al. Stage of chronic kidney disease predicts seroconversion after hepatitis B immunization: earlier is better. Am J Kidney Dis 2003; 42:1184–1192.
  9. Ghadiani MH, Besharati S, Mousavinasab N, Jalalzadeh M. Response rates to HB vaccine in CKD stages 3-4 and hemodialysis patients. J Res Med Sci 2012; 17:527–533.
  10. Jack AD, Hall AJ, Maine N, Mendy M, Whittle HC. What level of hepatitis B antibody is protective? J Infect Dis 1999; 179:489–492.
  11. Guidelines for vaccination in patients with chronic kidney disease. Indian J Nephrol 2016; 26(suppl 1):S15–S18.
  12. Somi MH, Hajipour B. Improving hepatitis B vaccine efficacy in end-stage renal diseases patients and role of adjuvants. ISRN Gastroenterol 2012; 2012:960413.
  13. Yousaf F, Gandham S, Galler M, Spinowitz B, Charytan C. Systematic review of the efficacy and safety of intradermal versus intramuscular hepatitis B vaccination in end-stage renal disease population unresponsive to primary vaccination series. Ren Fail 2015; 37:1080–1088.
Issue
Cleveland Clinic Journal of Medicine - 85(1)
Issue
Cleveland Clinic Journal of Medicine - 85(1)
Page Number
32-34
Page Number
32-34
Publications
Publications
Topics
Article Type
Display Headline
What is the hepatitis B vaccination regimen in chronic kidney disease?
Display Headline
What is the hepatitis B vaccination regimen in chronic kidney disease?
Legacy Keywords
hepatitis B, HBV, vaccination, chronic kidney disease, CKD, Engerix-B, Recombivax-HB, Kheng Yong Ong, Hong Yun Wong, Giat Yeng Khee
Legacy Keywords
hepatitis B, HBV, vaccination, chronic kidney disease, CKD, Engerix-B, Recombivax-HB, Kheng Yong Ong, Hong Yun Wong, Giat Yeng Khee
Sections
Disallow All Ads
Content Gating
No Gating (article Unlocked/Free)
Alternative CME
Disqus Comments
Default
Use ProPublica
Article PDF Media

Is it time to abandon fasting for routine lipid testing?

Article Type
Changed
Fri, 12/01/2017 - 09:57
Display Headline
Is it time to abandon fasting for routine lipid testing?

Yes. The time has come to change the way we think about fasting before routine lipid testing. We now have robust evidence supporting the routine use of nonfasting lipid testing. Fasting lipid testing is rarely needed, but may be considered for patients with very high triglycerides or before starting treatment in patients with genetic lipid disorders. For most patients, nonfasting lipid testing is appropriate: it is evidence-based, safe, valid, and convenient. More widespread adoption of this strategy by US healthcare providers would improve quality of care and patient and clinician satisfaction.

GUIDELINES HAVE CHANGED

In 2014, the US Department of Veterans Affairs practice guidelines recommended nonfasting lipid testing for cardiovascular risk assessment.1 Other recent clinical guidelines and expert consensus statements from Europe and Canada now also recommend nonfasting lipid testing for most routine clinical evaluations.

Physiologically, we spend most of our lives in the nonfasting state, yet fasting lipid testing was standard practice advocated by earlier clinical guidelines. The rationale for fasting before measuring lipids was to reduce variability and to allow for a more accurate derivation of the low-density lipoprotein cholesterol (LDL-C) concentration using the Friedewald formula. There was also concern that an increase in triglyceride concentrations after consuming a fatty meal would reduce the validity of the results. However, numerous studies have found that the increase in plasma triglycerides after normal food intake is much less than that during a fat-tolerance test, making this less of a concern for most patients.2,3

In addition, recent studies suggest that postprandial effects do not diminish and may even strengthen the risk associations of lipids with cardiovascular disease, in particular for triglycerides.4 Moreover, in certain patients, such as those with metabolic syndrome, diabetes mellitus, or certain genetic abnormalities, fasting can mask abnormalities in triglyceride-rich lipid metabolism, which may only be detected when triglycerides are measured in a nonfasting state. Nonfasting measurements may identify patients with elevated residual risk despite optimal guideline-based treatment.

Guidelines and recommendations that support nonfasting lipid testing
In 2016, a joint consensus statement of the European Atherosclerosis Society and the European Federation of Clinical Chemistry and Laboratory Medicine5 recommended nonfasting lipid testing as the new standard for lipid measurement, with fasting lipid testing considered for patients with triglyceride levels over 400 mg/dL (5 mmol/L). The statement also recommends that nonfasting triglyceride levels greater than or equal to 175 mg/dL (2 mmol/L) should be considered elevated as compared with the 150 mg/dL (1.7 mmol/L) traditionally used for fasting panels.

Recently published recommendations for nonfasting lipid testing for routine assessments are summarized in Table 1.1,5–11

EFFECTS OF THE POSTPRANDIAL STATE ON LIPID LEVELS AND RISK ASSESSMENT

A common concern for clinicians who do not routinely order nonfasting lipid testing is the potential variability in lipid levels and interpretation of these values for treatment decisions. But in most circumstances the differences between fasting and nonfasting measurements are small and are not clinically relevant. Differences in high-density lipoprotein  cholesterol (HDL-C) are negligible; slightly lower levels are seen (up to −8 mg/dL) for nonfasting total cholesterol, LDL-C, and non-HDL-C compared with fasting levels; and differences are modest (up to 25 mg/dL higher) for triglycerides.5 These data should reassure clinicians who rely on lipid levels to guide management decisions.9

Cardiovascular risk assessment

Current algorithms for assessing risk of cardiovascular disease use total cholesterol and HDL-C, not triglycerides or LDL-C. Hence, eating has no effect on the risk estimates.

For clinicians who prefer an absolute lipid target for managing risk in patients on lipid-modifying therapy, a nonfasting LDL-C or non-HDL-C (or apolipoprotein B) may be used. The non-HDL-C level is a better risk marker than LDL-C, particularly in patients with low LDL-C or with triglyceride levels of 200 mg/dL or higher.12 Treatment goals for non-HDL-C are 30 mg/dL higher than for LDL-C (fasting or nonfasting). In addition, for these patients with low LDL-C or high triglycerides, a new LDL-C calculation method has more consistent results for fasting and nonfasting values than the commonly used Friedewald calculation.12

 

 

EVIDENCE SUPPORTING NONFASTING LIPID TESTING

The adequacy of nonfasting lipid testing for general screening for cardiovascular disease has been verified in large prospective studies over the past several decades.2,13,14 These studies evaluated cardiovascular event and mortality rates and found consistent associations of nonfasting lipid levels with cardiovascular disease. Studies that included both fasting and nonfasting patient populations found similar or occasionally even greater cardiovascular risk associations for nonfasting lipid measurements (including for LDL-C and triglycerides) compared with fasting lipid measurements.

The Emerging Risk Factors Collaboration14 reviewed the data from 68 studies in more than 300,000 people and found that the relationship between lipid levels and incident cardiovascular events was just as strong when nonfasting lipid values were used. In fact, at least 3 large statin trials reviewed (a total of 43,000 people) used nonfasting lipids.14

Genetic studies using mendelian randomization have also linked nonfasting triglyceride levels (and remnant cholesterol) to an increased risk of cardiovascular events and of death from any cause.15,16

Therefore, the evidence overall suggests that nonfasting lipid measurements are acceptable with respect to risk assessment, and indeed may be preferred in most instances, especially in patients with an atherogenic metabolic milieu that may otherwise be masked by the fasting state.

OTHER BENEFITS OF NONFASTING LIPID TESTING

Nonfasting lipid panels are more economical and safer for certain groups, such as elderly or diabetic patients. A pilot study17 found that up to 27.1% of patients with diabetes reported experiencing a fasting-evoked hypoglycemic event en route to testing because of fasting for blood work. These events are vastly underreported and add to patient morbidity that can easily be avoided by adopting nonfasting lipid testing.

No study has assessed the cost-effectiveness of fasting vs nonfasting lipid testing. It is common for patients to present for their office appointment without having obtained a fasting lipid panel simply because they forgot to fast and were turned away by the laboratory. Thus, management decisions during the visit are often deferred, and patients must return to the laboratory and reschedule follow-up visits. This is inefficient, increases outpatient waiting times, and also potentially deprives others of access to needed care. Laboratory workflow can also suffer from an influx of early morning visits for fasting tests, decreasing system efficiency. Decreased efficiency in multiple levels of the healthcare system leads to increased costs, burden on healthcare providers, and decreased patient and physician satisfaction.

GETTING WITH THE GUIDELINES

The 2002 National Cholesterol Education Program expert panel report18 and the 2013 joint cholesterol guidelines of the American College of Cardiology and the American Heart Association9 both recommended that initial screening should involve fasting lipid testing, but they also allowed measuring nonfasting total cholesterol, HDL-C, and non-HDL-C.18 And internationally, there has been a shift in practice recommendations toward nonfasting lipids over the past 10 years (Table 1).

In 2014, the US Department of Veterans Affairs, the UK National Clinical Guideline Centre, and the Joint British Societies said that fasting is no longer needed for routine testing.10 In 2016, the European Atherosclerosis Society and the European Federation of Clinical Chemistry and Laboratory Medicine recommended nonfasting lipid testing as the standard of care and provided clinically useful cut points for both fasting and nonfasting lipid measurements.5

In most guidelines, the threshold for elevated nonfasting triglycerides was defined as 175 mg/dL (≥ 2 mmol/L) or greater, and this level has been validated prospectively in a large study of US women.5,19 Repeat measurement of fasting triglycerides may be considered when nonfasting levels are greater than  400 mg/dL,5 although there is no consensus in the guidelines regarding when or if fasting triglycerides should be remeasured. (In the Danish experience,5 only 10% of patients have required repeat fasting values). In addition, the 2016 Canadian Hypertension Education Program guidelines6 removed fasting as a requirement. The 2016 Canadian Cardiovascular Society dyslipidemia guidelines7 reported that nonfasting lipid testing is a suitable alternative to fasting. Furthermore, the most recent revision of the European Society of Cardiology dyslipidemia guidelines8 acknowledged that nonfasting lipid panels are acceptable for screening and management of patients without severe hypertriglyceridemia or those with extremely low LDL-C levels.

LIMITATIONS OF THE EVIDENCE

To date, no study has assessed the predictive value of fasting vs nonfasting lipid measurements in the same individuals, and there have been no randomized outcomes trials or cost-effectiveness analyses. Ethnic variations in lipoproteins and nonfasting status also need to be investigated as nonfasting lipid testing becomes more universally accepted.

TAKE-HOME POINTS

  • Robust evidence supports the routine use of nonfasting lipid testing, with fasting panels reserved potentially for patients with very high triglycerides and before starting treatment in those with genetic lipid disorders.
  • For most patients, nonfasting tests are evidence-based, safe, valid, and convenient.
  • More widespread adoption of this strategy by US healthcare providers would improve both quality of care and patient-clinician satisfaction.
References
  1. US Department of Veterans Affairs. VA/DoD Clinical Practice Guidelines: the management of dyslipidemia for cardiovascular risk reduction (lipids). 2014. www.healthquality.va.gov/guidelines/CD/lipids. Accessed October 18, 2017.
  2. Langsted A, Freiberg JJ, Nordestgaard BG. Fasting and nonfasting lipid levels: influence of normal food intake on lipids, lipoproteins, apolipoproteins, and cardiovascular risk prediction. Circulation 2008; 118:2047–2056.
  3. Langsted A, Nordestgaard BG. Nonfasting lipids, lipoproteins, and apolipoproteins in individuals with and without diabetes: 58 434 individuals from the Copenhagen General Population Study. Clin Chem 2011; 57:482–489.
  4. Rifai N, Young IS, Nordestgaard BG, et al. Nonfasting sample for the determination of routine lipid profile: is it an idea whose time has come? Clin Chem 2016; 62:428–435.
  5. Nordestgaard BG, Langsted A, Mora S, et al; European Atherosclerosis Society (EAS) and the European Federation of Clinical Chemistry and Laboratory Medicine (EFLM) joint consensus initiative. Fasting is not routinely required for determination of a lipid profile: clinical and laboratory implications including flagging at desirable concentration cut-points-a joint consensus statement from the European Atherosclerosis Society and European Federation of Clinical Chemistry and Laboratory Medicine. Eur Heart J 2016; 37:1944–1958.
  6. Leung AA, Nerenberg K, Daskalopoulou SS, et al; CHEP Guidelines Task Force. Hypertension Canada’s 2016 Canadian Hypertension Education Program guidelines for blood pressure measurement, diagnosis, assessment of risk, prevention, and treatment of hypertension. Can J Cardiol 2016; 32:569–588.
  7. Anderson TJ, Gregoire J, Pearson GJ, et al. 2016 Canadian Cardiovascular Society guidelines for the management of dyslipidemia for the prevention of cardiovascular disease in the adult. Can J Cardiol 2016; 32:1263–1282.
  8. Catapano AL, Graham I, De Backer G, et al; Authors/Task Force Members; Additional Contributor. 2016 ESC/EAS guidelines for the management of dyslipidaemias. Eur Heart J 2016; 37:2999–3058.
  9. Stone NJ, Robinson JG, Lichtenstein AH, et al; American College of Cardiology/American Heart Association Task Force on Practice Guidelines. 2013 ACC/AHA guideline on the treatment of blood cholesterol to reduce atherosclerotic cardiovascular risk in adults: a report of the American College of Cardiology/American Heart Association Task Force on Practice Guidelines. Circulation 2014; 129(suppl):S1–S45.
  10. National Institute for Health and Care Excellence (NICE). Cardiovascular disease: risk assessment and reduction, including lipid modification. Clinical guideline CG181. Published July 2014. Updated September 2016. www.nice.org.uk/guidance/cg181. Accessed October 18, 2017.
  11. Jellinger PS, Handelsman Y, Rosenblit PD, et al. American Association of Clinical Endocrinologists and American College of Endocrinology guidelines for management of dyslipidemia and prevention of cardiovascular disease. Endocr Pract 2017; 23(suppl 2):1–87.
  12. Martin SS, Blaha MJ, Elshazly MB, et al. Friedewald-estimated versus directly measured low-density lipoprotein cholesterol and treatment implications. J Am Coll Cardiol 2013; 62:732–739.
  13. Mora S, Rifai N, Buring JE, Ridker PM. Fasting compared with nonfasting lipids and apolipoproteins for predicting incident cardiovascular events. Circulation 2008; 118:993–1001.
  14. Emerging Risk Factors Collaboration; Di Angelantonio E, Sarwar N, Perry P, et al. Major lipids, apolipoproteins, and risk of vascular disease. JAMA 2009; 302:1993–2000.
  15. Varbo A, Benn M, Tybjaerg-Hansen A, Jorgensen AB, Frikke-Schmidt R, Nordestgaard BG. Remnant cholesterol as a causal risk factor for ischemic heart disease. J Am Coll Cardiol 2013; 61:427–436.
  16. Jorgensen AB, Frikke-Schmidt R, Nordestgaard BG, Tybjærg-Hansen A. Loss-of-function mutations in APOC3 and risk of ischemic vascular disease. N Engl J Med 2014; 371:32–41.
  17. Aldasouqi S, Corser W, Abela G, et al. Fasting for lipid profiles poses a high risk of hypoglycemia in patients with diabetes: a pilot prevalence study in clinical practice. Int J Clin Med 2016; 7:1653–1667.
  18. National Cholesterol Education Program (NCEP) Expert Panel on Detection, Evaluation, and Treatment of High Blood Cholesterol in Adults (Adult Treatment Panel III). Third report of the National Cholesterol Education Program (NCEP) Expert Panel on detection, evaluation, and treatment of high blood cholesterol in adults (Adult Treatment Panel III) final report. Circulation 2002; 106:3143–3421.
  19. White KT, Moorthy MV, Akinkuolie AO, et al. Identifying an optimal cutpoint for the diagnosis of hypertriglyceridemia in the nonfasting state. Clin Chem 2015; 61:1156–1163.
Article PDF
Author and Disclosure Information

Zareen Farukhi, MD
Center for Lipid Metabolomics, Division of Preventive Medicine, Department of Medicine, Brigham and Women’s Hospital, Harvard Medical School, Boston, MA

Samia Mora, MD, MHS
Center for Lipid Metabolomics, Division of Preventive Medicine, Division of Cardiovascular Medicine,
Department of Medicine, Brigham and Women’s Hospital, Harvard Medical School, Boston, MA

Address: Samia Mora, MD, MHS, Center for Lipid Metabolomics, Division of Preventive Medicine, Brigham and Women’s Hospital, Harvard Medical School, 900 Commonwealth Avenue E, Boston, MA 02215; smora@partners.org

Dr. Farukhi was supported by the National Heart, Lung, and Blood Institute (T32 HL007575).

Issue
Cleveland Clinic Journal of Medicine - 84(12)
Publications
Topics
Page Number
919-922
Legacy Keywords
lipids, testing, phlebotomy, cholesterol, LDL-C, fasting, blood test, triglycerides, Zareen Farukhi, Samia Mora
Sections
Author and Disclosure Information

Zareen Farukhi, MD
Center for Lipid Metabolomics, Division of Preventive Medicine, Department of Medicine, Brigham and Women’s Hospital, Harvard Medical School, Boston, MA

Samia Mora, MD, MHS
Center for Lipid Metabolomics, Division of Preventive Medicine, Division of Cardiovascular Medicine,
Department of Medicine, Brigham and Women’s Hospital, Harvard Medical School, Boston, MA

Address: Samia Mora, MD, MHS, Center for Lipid Metabolomics, Division of Preventive Medicine, Brigham and Women’s Hospital, Harvard Medical School, 900 Commonwealth Avenue E, Boston, MA 02215; smora@partners.org

Dr. Farukhi was supported by the National Heart, Lung, and Blood Institute (T32 HL007575).

Author and Disclosure Information

Zareen Farukhi, MD
Center for Lipid Metabolomics, Division of Preventive Medicine, Department of Medicine, Brigham and Women’s Hospital, Harvard Medical School, Boston, MA

Samia Mora, MD, MHS
Center for Lipid Metabolomics, Division of Preventive Medicine, Division of Cardiovascular Medicine,
Department of Medicine, Brigham and Women’s Hospital, Harvard Medical School, Boston, MA

Address: Samia Mora, MD, MHS, Center for Lipid Metabolomics, Division of Preventive Medicine, Brigham and Women’s Hospital, Harvard Medical School, 900 Commonwealth Avenue E, Boston, MA 02215; smora@partners.org

Dr. Farukhi was supported by the National Heart, Lung, and Blood Institute (T32 HL007575).

Article PDF
Article PDF
Related Articles

Yes. The time has come to change the way we think about fasting before routine lipid testing. We now have robust evidence supporting the routine use of nonfasting lipid testing. Fasting lipid testing is rarely needed, but may be considered for patients with very high triglycerides or before starting treatment in patients with genetic lipid disorders. For most patients, nonfasting lipid testing is appropriate: it is evidence-based, safe, valid, and convenient. More widespread adoption of this strategy by US healthcare providers would improve quality of care and patient and clinician satisfaction.

GUIDELINES HAVE CHANGED

In 2014, the US Department of Veterans Affairs practice guidelines recommended nonfasting lipid testing for cardiovascular risk assessment.1 Other recent clinical guidelines and expert consensus statements from Europe and Canada now also recommend nonfasting lipid testing for most routine clinical evaluations.

Physiologically, we spend most of our lives in the nonfasting state, yet fasting lipid testing was standard practice advocated by earlier clinical guidelines. The rationale for fasting before measuring lipids was to reduce variability and to allow for a more accurate derivation of the low-density lipoprotein cholesterol (LDL-C) concentration using the Friedewald formula. There was also concern that an increase in triglyceride concentrations after consuming a fatty meal would reduce the validity of the results. However, numerous studies have found that the increase in plasma triglycerides after normal food intake is much less than that during a fat-tolerance test, making this less of a concern for most patients.2,3

In addition, recent studies suggest that postprandial effects do not diminish and may even strengthen the risk associations of lipids with cardiovascular disease, in particular for triglycerides.4 Moreover, in certain patients, such as those with metabolic syndrome, diabetes mellitus, or certain genetic abnormalities, fasting can mask abnormalities in triglyceride-rich lipid metabolism, which may only be detected when triglycerides are measured in a nonfasting state. Nonfasting measurements may identify patients with elevated residual risk despite optimal guideline-based treatment.

Guidelines and recommendations that support nonfasting lipid testing
In 2016, a joint consensus statement of the European Atherosclerosis Society and the European Federation of Clinical Chemistry and Laboratory Medicine5 recommended nonfasting lipid testing as the new standard for lipid measurement, with fasting lipid testing considered for patients with triglyceride levels over 400 mg/dL (5 mmol/L). The statement also recommends that nonfasting triglyceride levels greater than or equal to 175 mg/dL (2 mmol/L) should be considered elevated as compared with the 150 mg/dL (1.7 mmol/L) traditionally used for fasting panels.

Recently published recommendations for nonfasting lipid testing for routine assessments are summarized in Table 1.1,5–11

EFFECTS OF THE POSTPRANDIAL STATE ON LIPID LEVELS AND RISK ASSESSMENT

A common concern for clinicians who do not routinely order nonfasting lipid testing is the potential variability in lipid levels and interpretation of these values for treatment decisions. But in most circumstances the differences between fasting and nonfasting measurements are small and are not clinically relevant. Differences in high-density lipoprotein  cholesterol (HDL-C) are negligible; slightly lower levels are seen (up to −8 mg/dL) for nonfasting total cholesterol, LDL-C, and non-HDL-C compared with fasting levels; and differences are modest (up to 25 mg/dL higher) for triglycerides.5 These data should reassure clinicians who rely on lipid levels to guide management decisions.9

Cardiovascular risk assessment

Current algorithms for assessing risk of cardiovascular disease use total cholesterol and HDL-C, not triglycerides or LDL-C. Hence, eating has no effect on the risk estimates.

For clinicians who prefer an absolute lipid target for managing risk in patients on lipid-modifying therapy, a nonfasting LDL-C or non-HDL-C (or apolipoprotein B) may be used. The non-HDL-C level is a better risk marker than LDL-C, particularly in patients with low LDL-C or with triglyceride levels of 200 mg/dL or higher.12 Treatment goals for non-HDL-C are 30 mg/dL higher than for LDL-C (fasting or nonfasting). In addition, for these patients with low LDL-C or high triglycerides, a new LDL-C calculation method has more consistent results for fasting and nonfasting values than the commonly used Friedewald calculation.12

 

 

EVIDENCE SUPPORTING NONFASTING LIPID TESTING

The adequacy of nonfasting lipid testing for general screening for cardiovascular disease has been verified in large prospective studies over the past several decades.2,13,14 These studies evaluated cardiovascular event and mortality rates and found consistent associations of nonfasting lipid levels with cardiovascular disease. Studies that included both fasting and nonfasting patient populations found similar or occasionally even greater cardiovascular risk associations for nonfasting lipid measurements (including for LDL-C and triglycerides) compared with fasting lipid measurements.

The Emerging Risk Factors Collaboration14 reviewed the data from 68 studies in more than 300,000 people and found that the relationship between lipid levels and incident cardiovascular events was just as strong when nonfasting lipid values were used. In fact, at least 3 large statin trials reviewed (a total of 43,000 people) used nonfasting lipids.14

Genetic studies using mendelian randomization have also linked nonfasting triglyceride levels (and remnant cholesterol) to an increased risk of cardiovascular events and of death from any cause.15,16

Therefore, the evidence overall suggests that nonfasting lipid measurements are acceptable with respect to risk assessment, and indeed may be preferred in most instances, especially in patients with an atherogenic metabolic milieu that may otherwise be masked by the fasting state.

OTHER BENEFITS OF NONFASTING LIPID TESTING

Nonfasting lipid panels are more economical and safer for certain groups, such as elderly or diabetic patients. A pilot study17 found that up to 27.1% of patients with diabetes reported experiencing a fasting-evoked hypoglycemic event en route to testing because of fasting for blood work. These events are vastly underreported and add to patient morbidity that can easily be avoided by adopting nonfasting lipid testing.

No study has assessed the cost-effectiveness of fasting vs nonfasting lipid testing. It is common for patients to present for their office appointment without having obtained a fasting lipid panel simply because they forgot to fast and were turned away by the laboratory. Thus, management decisions during the visit are often deferred, and patients must return to the laboratory and reschedule follow-up visits. This is inefficient, increases outpatient waiting times, and also potentially deprives others of access to needed care. Laboratory workflow can also suffer from an influx of early morning visits for fasting tests, decreasing system efficiency. Decreased efficiency in multiple levels of the healthcare system leads to increased costs, burden on healthcare providers, and decreased patient and physician satisfaction.

GETTING WITH THE GUIDELINES

The 2002 National Cholesterol Education Program expert panel report18 and the 2013 joint cholesterol guidelines of the American College of Cardiology and the American Heart Association9 both recommended that initial screening should involve fasting lipid testing, but they also allowed measuring nonfasting total cholesterol, HDL-C, and non-HDL-C.18 And internationally, there has been a shift in practice recommendations toward nonfasting lipids over the past 10 years (Table 1).

In 2014, the US Department of Veterans Affairs, the UK National Clinical Guideline Centre, and the Joint British Societies said that fasting is no longer needed for routine testing.10 In 2016, the European Atherosclerosis Society and the European Federation of Clinical Chemistry and Laboratory Medicine recommended nonfasting lipid testing as the standard of care and provided clinically useful cut points for both fasting and nonfasting lipid measurements.5

In most guidelines, the threshold for elevated nonfasting triglycerides was defined as 175 mg/dL (≥ 2 mmol/L) or greater, and this level has been validated prospectively in a large study of US women.5,19 Repeat measurement of fasting triglycerides may be considered when nonfasting levels are greater than  400 mg/dL,5 although there is no consensus in the guidelines regarding when or if fasting triglycerides should be remeasured. (In the Danish experience,5 only 10% of patients have required repeat fasting values). In addition, the 2016 Canadian Hypertension Education Program guidelines6 removed fasting as a requirement. The 2016 Canadian Cardiovascular Society dyslipidemia guidelines7 reported that nonfasting lipid testing is a suitable alternative to fasting. Furthermore, the most recent revision of the European Society of Cardiology dyslipidemia guidelines8 acknowledged that nonfasting lipid panels are acceptable for screening and management of patients without severe hypertriglyceridemia or those with extremely low LDL-C levels.

LIMITATIONS OF THE EVIDENCE

To date, no study has assessed the predictive value of fasting vs nonfasting lipid measurements in the same individuals, and there have been no randomized outcomes trials or cost-effectiveness analyses. Ethnic variations in lipoproteins and nonfasting status also need to be investigated as nonfasting lipid testing becomes more universally accepted.

TAKE-HOME POINTS

  • Robust evidence supports the routine use of nonfasting lipid testing, with fasting panels reserved potentially for patients with very high triglycerides and before starting treatment in those with genetic lipid disorders.
  • For most patients, nonfasting tests are evidence-based, safe, valid, and convenient.
  • More widespread adoption of this strategy by US healthcare providers would improve both quality of care and patient-clinician satisfaction.

Yes. The time has come to change the way we think about fasting before routine lipid testing. We now have robust evidence supporting the routine use of nonfasting lipid testing. Fasting lipid testing is rarely needed, but may be considered for patients with very high triglycerides or before starting treatment in patients with genetic lipid disorders. For most patients, nonfasting lipid testing is appropriate: it is evidence-based, safe, valid, and convenient. More widespread adoption of this strategy by US healthcare providers would improve quality of care and patient and clinician satisfaction.

GUIDELINES HAVE CHANGED

In 2014, the US Department of Veterans Affairs practice guidelines recommended nonfasting lipid testing for cardiovascular risk assessment.1 Other recent clinical guidelines and expert consensus statements from Europe and Canada now also recommend nonfasting lipid testing for most routine clinical evaluations.

Physiologically, we spend most of our lives in the nonfasting state, yet fasting lipid testing was standard practice advocated by earlier clinical guidelines. The rationale for fasting before measuring lipids was to reduce variability and to allow for a more accurate derivation of the low-density lipoprotein cholesterol (LDL-C) concentration using the Friedewald formula. There was also concern that an increase in triglyceride concentrations after consuming a fatty meal would reduce the validity of the results. However, numerous studies have found that the increase in plasma triglycerides after normal food intake is much less than that during a fat-tolerance test, making this less of a concern for most patients.2,3

In addition, recent studies suggest that postprandial effects do not diminish and may even strengthen the risk associations of lipids with cardiovascular disease, in particular for triglycerides.4 Moreover, in certain patients, such as those with metabolic syndrome, diabetes mellitus, or certain genetic abnormalities, fasting can mask abnormalities in triglyceride-rich lipid metabolism, which may only be detected when triglycerides are measured in a nonfasting state. Nonfasting measurements may identify patients with elevated residual risk despite optimal guideline-based treatment.

Guidelines and recommendations that support nonfasting lipid testing
In 2016, a joint consensus statement of the European Atherosclerosis Society and the European Federation of Clinical Chemistry and Laboratory Medicine5 recommended nonfasting lipid testing as the new standard for lipid measurement, with fasting lipid testing considered for patients with triglyceride levels over 400 mg/dL (5 mmol/L). The statement also recommends that nonfasting triglyceride levels greater than or equal to 175 mg/dL (2 mmol/L) should be considered elevated as compared with the 150 mg/dL (1.7 mmol/L) traditionally used for fasting panels.

Recently published recommendations for nonfasting lipid testing for routine assessments are summarized in Table 1.1,5–11

EFFECTS OF THE POSTPRANDIAL STATE ON LIPID LEVELS AND RISK ASSESSMENT

A common concern for clinicians who do not routinely order nonfasting lipid testing is the potential variability in lipid levels and interpretation of these values for treatment decisions. But in most circumstances the differences between fasting and nonfasting measurements are small and are not clinically relevant. Differences in high-density lipoprotein  cholesterol (HDL-C) are negligible; slightly lower levels are seen (up to −8 mg/dL) for nonfasting total cholesterol, LDL-C, and non-HDL-C compared with fasting levels; and differences are modest (up to 25 mg/dL higher) for triglycerides.5 These data should reassure clinicians who rely on lipid levels to guide management decisions.9

Cardiovascular risk assessment

Current algorithms for assessing risk of cardiovascular disease use total cholesterol and HDL-C, not triglycerides or LDL-C. Hence, eating has no effect on the risk estimates.

For clinicians who prefer an absolute lipid target for managing risk in patients on lipid-modifying therapy, a nonfasting LDL-C or non-HDL-C (or apolipoprotein B) may be used. The non-HDL-C level is a better risk marker than LDL-C, particularly in patients with low LDL-C or with triglyceride levels of 200 mg/dL or higher.12 Treatment goals for non-HDL-C are 30 mg/dL higher than for LDL-C (fasting or nonfasting). In addition, for these patients with low LDL-C or high triglycerides, a new LDL-C calculation method has more consistent results for fasting and nonfasting values than the commonly used Friedewald calculation.12

 

 

EVIDENCE SUPPORTING NONFASTING LIPID TESTING

The adequacy of nonfasting lipid testing for general screening for cardiovascular disease has been verified in large prospective studies over the past several decades.2,13,14 These studies evaluated cardiovascular event and mortality rates and found consistent associations of nonfasting lipid levels with cardiovascular disease. Studies that included both fasting and nonfasting patient populations found similar or occasionally even greater cardiovascular risk associations for nonfasting lipid measurements (including for LDL-C and triglycerides) compared with fasting lipid measurements.

The Emerging Risk Factors Collaboration14 reviewed the data from 68 studies in more than 300,000 people and found that the relationship between lipid levels and incident cardiovascular events was just as strong when nonfasting lipid values were used. In fact, at least 3 large statin trials reviewed (a total of 43,000 people) used nonfasting lipids.14

Genetic studies using mendelian randomization have also linked nonfasting triglyceride levels (and remnant cholesterol) to an increased risk of cardiovascular events and of death from any cause.15,16

Therefore, the evidence overall suggests that nonfasting lipid measurements are acceptable with respect to risk assessment, and indeed may be preferred in most instances, especially in patients with an atherogenic metabolic milieu that may otherwise be masked by the fasting state.

OTHER BENEFITS OF NONFASTING LIPID TESTING

Nonfasting lipid panels are more economical and safer for certain groups, such as elderly or diabetic patients. A pilot study17 found that up to 27.1% of patients with diabetes reported experiencing a fasting-evoked hypoglycemic event en route to testing because of fasting for blood work. These events are vastly underreported and add to patient morbidity that can easily be avoided by adopting nonfasting lipid testing.

No study has assessed the cost-effectiveness of fasting vs nonfasting lipid testing. It is common for patients to present for their office appointment without having obtained a fasting lipid panel simply because they forgot to fast and were turned away by the laboratory. Thus, management decisions during the visit are often deferred, and patients must return to the laboratory and reschedule follow-up visits. This is inefficient, increases outpatient waiting times, and also potentially deprives others of access to needed care. Laboratory workflow can also suffer from an influx of early morning visits for fasting tests, decreasing system efficiency. Decreased efficiency in multiple levels of the healthcare system leads to increased costs, burden on healthcare providers, and decreased patient and physician satisfaction.

GETTING WITH THE GUIDELINES

The 2002 National Cholesterol Education Program expert panel report18 and the 2013 joint cholesterol guidelines of the American College of Cardiology and the American Heart Association9 both recommended that initial screening should involve fasting lipid testing, but they also allowed measuring nonfasting total cholesterol, HDL-C, and non-HDL-C.18 And internationally, there has been a shift in practice recommendations toward nonfasting lipids over the past 10 years (Table 1).

In 2014, the US Department of Veterans Affairs, the UK National Clinical Guideline Centre, and the Joint British Societies said that fasting is no longer needed for routine testing.10 In 2016, the European Atherosclerosis Society and the European Federation of Clinical Chemistry and Laboratory Medicine recommended nonfasting lipid testing as the standard of care and provided clinically useful cut points for both fasting and nonfasting lipid measurements.5

In most guidelines, the threshold for elevated nonfasting triglycerides was defined as 175 mg/dL (≥ 2 mmol/L) or greater, and this level has been validated prospectively in a large study of US women.5,19 Repeat measurement of fasting triglycerides may be considered when nonfasting levels are greater than  400 mg/dL,5 although there is no consensus in the guidelines regarding when or if fasting triglycerides should be remeasured. (In the Danish experience,5 only 10% of patients have required repeat fasting values). In addition, the 2016 Canadian Hypertension Education Program guidelines6 removed fasting as a requirement. The 2016 Canadian Cardiovascular Society dyslipidemia guidelines7 reported that nonfasting lipid testing is a suitable alternative to fasting. Furthermore, the most recent revision of the European Society of Cardiology dyslipidemia guidelines8 acknowledged that nonfasting lipid panels are acceptable for screening and management of patients without severe hypertriglyceridemia or those with extremely low LDL-C levels.

LIMITATIONS OF THE EVIDENCE

To date, no study has assessed the predictive value of fasting vs nonfasting lipid measurements in the same individuals, and there have been no randomized outcomes trials or cost-effectiveness analyses. Ethnic variations in lipoproteins and nonfasting status also need to be investigated as nonfasting lipid testing becomes more universally accepted.

TAKE-HOME POINTS

  • Robust evidence supports the routine use of nonfasting lipid testing, with fasting panels reserved potentially for patients with very high triglycerides and before starting treatment in those with genetic lipid disorders.
  • For most patients, nonfasting tests are evidence-based, safe, valid, and convenient.
  • More widespread adoption of this strategy by US healthcare providers would improve both quality of care and patient-clinician satisfaction.
References
  1. US Department of Veterans Affairs. VA/DoD Clinical Practice Guidelines: the management of dyslipidemia for cardiovascular risk reduction (lipids). 2014. www.healthquality.va.gov/guidelines/CD/lipids. Accessed October 18, 2017.
  2. Langsted A, Freiberg JJ, Nordestgaard BG. Fasting and nonfasting lipid levels: influence of normal food intake on lipids, lipoproteins, apolipoproteins, and cardiovascular risk prediction. Circulation 2008; 118:2047–2056.
  3. Langsted A, Nordestgaard BG. Nonfasting lipids, lipoproteins, and apolipoproteins in individuals with and without diabetes: 58 434 individuals from the Copenhagen General Population Study. Clin Chem 2011; 57:482–489.
  4. Rifai N, Young IS, Nordestgaard BG, et al. Nonfasting sample for the determination of routine lipid profile: is it an idea whose time has come? Clin Chem 2016; 62:428–435.
  5. Nordestgaard BG, Langsted A, Mora S, et al; European Atherosclerosis Society (EAS) and the European Federation of Clinical Chemistry and Laboratory Medicine (EFLM) joint consensus initiative. Fasting is not routinely required for determination of a lipid profile: clinical and laboratory implications including flagging at desirable concentration cut-points-a joint consensus statement from the European Atherosclerosis Society and European Federation of Clinical Chemistry and Laboratory Medicine. Eur Heart J 2016; 37:1944–1958.
  6. Leung AA, Nerenberg K, Daskalopoulou SS, et al; CHEP Guidelines Task Force. Hypertension Canada’s 2016 Canadian Hypertension Education Program guidelines for blood pressure measurement, diagnosis, assessment of risk, prevention, and treatment of hypertension. Can J Cardiol 2016; 32:569–588.
  7. Anderson TJ, Gregoire J, Pearson GJ, et al. 2016 Canadian Cardiovascular Society guidelines for the management of dyslipidemia for the prevention of cardiovascular disease in the adult. Can J Cardiol 2016; 32:1263–1282.
  8. Catapano AL, Graham I, De Backer G, et al; Authors/Task Force Members; Additional Contributor. 2016 ESC/EAS guidelines for the management of dyslipidaemias. Eur Heart J 2016; 37:2999–3058.
  9. Stone NJ, Robinson JG, Lichtenstein AH, et al; American College of Cardiology/American Heart Association Task Force on Practice Guidelines. 2013 ACC/AHA guideline on the treatment of blood cholesterol to reduce atherosclerotic cardiovascular risk in adults: a report of the American College of Cardiology/American Heart Association Task Force on Practice Guidelines. Circulation 2014; 129(suppl):S1–S45.
  10. National Institute for Health and Care Excellence (NICE). Cardiovascular disease: risk assessment and reduction, including lipid modification. Clinical guideline CG181. Published July 2014. Updated September 2016. www.nice.org.uk/guidance/cg181. Accessed October 18, 2017.
  11. Jellinger PS, Handelsman Y, Rosenblit PD, et al. American Association of Clinical Endocrinologists and American College of Endocrinology guidelines for management of dyslipidemia and prevention of cardiovascular disease. Endocr Pract 2017; 23(suppl 2):1–87.
  12. Martin SS, Blaha MJ, Elshazly MB, et al. Friedewald-estimated versus directly measured low-density lipoprotein cholesterol and treatment implications. J Am Coll Cardiol 2013; 62:732–739.
  13. Mora S, Rifai N, Buring JE, Ridker PM. Fasting compared with nonfasting lipids and apolipoproteins for predicting incident cardiovascular events. Circulation 2008; 118:993–1001.
  14. Emerging Risk Factors Collaboration; Di Angelantonio E, Sarwar N, Perry P, et al. Major lipids, apolipoproteins, and risk of vascular disease. JAMA 2009; 302:1993–2000.
  15. Varbo A, Benn M, Tybjaerg-Hansen A, Jorgensen AB, Frikke-Schmidt R, Nordestgaard BG. Remnant cholesterol as a causal risk factor for ischemic heart disease. J Am Coll Cardiol 2013; 61:427–436.
  16. Jorgensen AB, Frikke-Schmidt R, Nordestgaard BG, Tybjærg-Hansen A. Loss-of-function mutations in APOC3 and risk of ischemic vascular disease. N Engl J Med 2014; 371:32–41.
  17. Aldasouqi S, Corser W, Abela G, et al. Fasting for lipid profiles poses a high risk of hypoglycemia in patients with diabetes: a pilot prevalence study in clinical practice. Int J Clin Med 2016; 7:1653–1667.
  18. National Cholesterol Education Program (NCEP) Expert Panel on Detection, Evaluation, and Treatment of High Blood Cholesterol in Adults (Adult Treatment Panel III). Third report of the National Cholesterol Education Program (NCEP) Expert Panel on detection, evaluation, and treatment of high blood cholesterol in adults (Adult Treatment Panel III) final report. Circulation 2002; 106:3143–3421.
  19. White KT, Moorthy MV, Akinkuolie AO, et al. Identifying an optimal cutpoint for the diagnosis of hypertriglyceridemia in the nonfasting state. Clin Chem 2015; 61:1156–1163.
References
  1. US Department of Veterans Affairs. VA/DoD Clinical Practice Guidelines: the management of dyslipidemia for cardiovascular risk reduction (lipids). 2014. www.healthquality.va.gov/guidelines/CD/lipids. Accessed October 18, 2017.
  2. Langsted A, Freiberg JJ, Nordestgaard BG. Fasting and nonfasting lipid levels: influence of normal food intake on lipids, lipoproteins, apolipoproteins, and cardiovascular risk prediction. Circulation 2008; 118:2047–2056.
  3. Langsted A, Nordestgaard BG. Nonfasting lipids, lipoproteins, and apolipoproteins in individuals with and without diabetes: 58 434 individuals from the Copenhagen General Population Study. Clin Chem 2011; 57:482–489.
  4. Rifai N, Young IS, Nordestgaard BG, et al. Nonfasting sample for the determination of routine lipid profile: is it an idea whose time has come? Clin Chem 2016; 62:428–435.
  5. Nordestgaard BG, Langsted A, Mora S, et al; European Atherosclerosis Society (EAS) and the European Federation of Clinical Chemistry and Laboratory Medicine (EFLM) joint consensus initiative. Fasting is not routinely required for determination of a lipid profile: clinical and laboratory implications including flagging at desirable concentration cut-points-a joint consensus statement from the European Atherosclerosis Society and European Federation of Clinical Chemistry and Laboratory Medicine. Eur Heart J 2016; 37:1944–1958.
  6. Leung AA, Nerenberg K, Daskalopoulou SS, et al; CHEP Guidelines Task Force. Hypertension Canada’s 2016 Canadian Hypertension Education Program guidelines for blood pressure measurement, diagnosis, assessment of risk, prevention, and treatment of hypertension. Can J Cardiol 2016; 32:569–588.
  7. Anderson TJ, Gregoire J, Pearson GJ, et al. 2016 Canadian Cardiovascular Society guidelines for the management of dyslipidemia for the prevention of cardiovascular disease in the adult. Can J Cardiol 2016; 32:1263–1282.
  8. Catapano AL, Graham I, De Backer G, et al; Authors/Task Force Members; Additional Contributor. 2016 ESC/EAS guidelines for the management of dyslipidaemias. Eur Heart J 2016; 37:2999–3058.
  9. Stone NJ, Robinson JG, Lichtenstein AH, et al; American College of Cardiology/American Heart Association Task Force on Practice Guidelines. 2013 ACC/AHA guideline on the treatment of blood cholesterol to reduce atherosclerotic cardiovascular risk in adults: a report of the American College of Cardiology/American Heart Association Task Force on Practice Guidelines. Circulation 2014; 129(suppl):S1–S45.
  10. National Institute for Health and Care Excellence (NICE). Cardiovascular disease: risk assessment and reduction, including lipid modification. Clinical guideline CG181. Published July 2014. Updated September 2016. www.nice.org.uk/guidance/cg181. Accessed October 18, 2017.
  11. Jellinger PS, Handelsman Y, Rosenblit PD, et al. American Association of Clinical Endocrinologists and American College of Endocrinology guidelines for management of dyslipidemia and prevention of cardiovascular disease. Endocr Pract 2017; 23(suppl 2):1–87.
  12. Martin SS, Blaha MJ, Elshazly MB, et al. Friedewald-estimated versus directly measured low-density lipoprotein cholesterol and treatment implications. J Am Coll Cardiol 2013; 62:732–739.
  13. Mora S, Rifai N, Buring JE, Ridker PM. Fasting compared with nonfasting lipids and apolipoproteins for predicting incident cardiovascular events. Circulation 2008; 118:993–1001.
  14. Emerging Risk Factors Collaboration; Di Angelantonio E, Sarwar N, Perry P, et al. Major lipids, apolipoproteins, and risk of vascular disease. JAMA 2009; 302:1993–2000.
  15. Varbo A, Benn M, Tybjaerg-Hansen A, Jorgensen AB, Frikke-Schmidt R, Nordestgaard BG. Remnant cholesterol as a causal risk factor for ischemic heart disease. J Am Coll Cardiol 2013; 61:427–436.
  16. Jorgensen AB, Frikke-Schmidt R, Nordestgaard BG, Tybjærg-Hansen A. Loss-of-function mutations in APOC3 and risk of ischemic vascular disease. N Engl J Med 2014; 371:32–41.
  17. Aldasouqi S, Corser W, Abela G, et al. Fasting for lipid profiles poses a high risk of hypoglycemia in patients with diabetes: a pilot prevalence study in clinical practice. Int J Clin Med 2016; 7:1653–1667.
  18. National Cholesterol Education Program (NCEP) Expert Panel on Detection, Evaluation, and Treatment of High Blood Cholesterol in Adults (Adult Treatment Panel III). Third report of the National Cholesterol Education Program (NCEP) Expert Panel on detection, evaluation, and treatment of high blood cholesterol in adults (Adult Treatment Panel III) final report. Circulation 2002; 106:3143–3421.
  19. White KT, Moorthy MV, Akinkuolie AO, et al. Identifying an optimal cutpoint for the diagnosis of hypertriglyceridemia in the nonfasting state. Clin Chem 2015; 61:1156–1163.
Issue
Cleveland Clinic Journal of Medicine - 84(12)
Issue
Cleveland Clinic Journal of Medicine - 84(12)
Page Number
919-922
Page Number
919-922
Publications
Publications
Topics
Article Type
Display Headline
Is it time to abandon fasting for routine lipid testing?
Display Headline
Is it time to abandon fasting for routine lipid testing?
Legacy Keywords
lipids, testing, phlebotomy, cholesterol, LDL-C, fasting, blood test, triglycerides, Zareen Farukhi, Samia Mora
Legacy Keywords
lipids, testing, phlebotomy, cholesterol, LDL-C, fasting, blood test, triglycerides, Zareen Farukhi, Samia Mora
Sections
Disallow All Ads
Content Gating
No Gating (article Unlocked/Free)
Alternative CME
Disqus Comments
Default
Article PDF Media